Sunteți pe pagina 1din 94

Copyright © 2014 Delhi Academy of Medical Sciences, All Rights Reserved.

1/94
Test Information
Test Name SWTS-ANATOMY-2017(MDMS) Total Questions 200

Test Type Examination Difficulty Level Difficult

Total Marks 600 Duration 180minutes

Test Question Language:- ENGLISH

(1). Complete transaction of the presacral nerve is difficult because the nerve :

a. Overlies the middle sacral vessels and is usually composed of two or three incompletely fused trunks

b. Is a subsidiary of the celiac plexus and is located along the inferior mesenteric artery

c. Is part of the ovarian plexus, which transverses the infundubulopelvic ligament

d. Is Frankenhauser’s ganglion, which also innervates the uterus and vagina.

Solution. (a) Overlies the middle sacral vessels and is usually composed of
two or three incompletely fused trunks
Ref:Novak’s Gynecology-89
Sol:
- The presacral neurectomy is an operation in which the superior hypogastric plexus is divided and resected in order to interrupt sensory
fibers from the uterus and cervix, and has been associated with relief of dysmenorrheal secondary to endometriosis in about 50 to 75% of
cases.
- Because there are several branches and their locations vary, and because other nerves supply these structures, the resection of these
nerves does not uniformly relieve the pain.

Correct Answer. a

Copyright © 2014 Delhi Academy of Medical Sciences, All Rights Reserved. 2/94
(2). A 25-year-old woman has experienced headaches for the past 4 months. She has a lesion producing a compression at the point identified
in Image
A. This is most likely to result in which of the following findings?

a. Decreased hearing

b. Cushing disease

c. Impairment of upward gaze

d. Bitemporal hemianopsia

Solution. (d) Bitemporal hemianopsia


Ref:Read the text below
Sol :
This is the center of the optic chiasm, and a mass such as a pituitary
adenoma in the sella turcica below could impinge upon the chiasm, with
manifestation of loss of the visual fields laterally.

Correct Answer. d

(3). Septum transversum gives rise to all of the following except

a. Lesser omentum

b. Coronary ligament

c. Ligament teres hepatitis

d. Falciparum ligament

Solution. (b) Coronary ligament


Ref:Gray’s anatomy - 1168
Sol:
- The coronary ligament is formed by the reflection of the peritoneum from the diaphragm onto the posterior surfaces of the right lobe of
the liver

Correct Answer. b

Copyright © 2014 Delhi Academy of Medical Sciences, All Rights Reserved. 3/94
(4). Tensor Tympani issupplied by

a. Facial

b. Glossopharyngeal

c. Trigeminal

d. Vagus

Solution. (c) Trigeminal


Ref: Langman's Medical Embryology 10thed pg 258
Sol:
Derivatives of the Pharyngeal Arches and Their Innervation

Correct Answer. c

Copyright © 2014 Delhi Academy of Medical Sciences, All Rights Reserved. 4/94
(5). The epididymis is located on the posterior aspect of which of the following structures?

a. Urinary bladder

b. Prostate

c. Testis

d. Ovary

Solution. (c) Testis


Ref: Essential Clinical Anatomy by Moore p 201.
Sol:
- The epididymis lies on the posterior surface of the testis, which is covered by the tunica vaginalis except at its posterior margin

Correct Answer. c

(6). The testes are covered by a tough fibrous coat known as the

a. Cremaster fascia

b. Tunica albuginea

c. Gubernaculum

d. Tunica dartos

Solution. (b) Tunica albuginea


Ref:Essential Clinical Anatomy by Moore p 201
Sol:
- The testes are covered with a tough fibrous coat know as the tunica albuginea

Correct Answer. b

Copyright © 2014 Delhi Academy of Medical Sciences, All Rights Reserved. 5/94
(7). True about superficial palmar arch is:

a. Direct continuation of radial artery

b. Lies deep to palmar aponeurosis

c. Integrity is tested by adson’s test

d. Supplies the metacarpals

Solution. (b) Lies deep to palmar aponeurosis


Ref.:Read the text below
Sol :

Correct Answer. b

Copyright © 2014 Delhi Academy of Medical Sciences, All Rights Reserved. 6/94
(8). The midpalmar space extends between?

a. Flexor retinaculum to proximal transverse palmar crease

b. Proximal transverse palmar crease to distal palmar crease

c. Flexor retinaculum to distal palmar crease

d. Proximal transverse palmar crease to metacarpophalangealjoint.

Solution. (c) Flexor retinaculum to distal palmar crease


Ref.:Read the text below
Sol :

Correct Answer. c

Copyright © 2014 Delhi Academy of Medical Sciences, All Rights Reserved. 7/94
(9). “Hour glass swelling” of hand develops in infection of:

a. Radial bursa

b. Ulnar bursa

c. Pulp spaces

d. Deep palmar space

Solution. (b) Ulnar bursa


Ref.:Read the text below
Sol :
Ulnar bursa
- Contains flexor digitorum superficialis & profundus tendon Communications
- Proximally: Synovial sheath of little finger
- Distally : Forearm space of Parona
- Laterally : Radial bursa
- Infection of this bursa occurs secondary to infection of little finger
Hour Glass Swelling (one swelling in palm & other in distal part of foream) is produced due to its communication with the forearm space.
Drained by incision along the lateral border of hypothenar eminence
- In ulnar bursitis, the area of maximum tenderness is elicited in the region between the two palmar creases (kanavel’s sign)

Correct Answer. b

(10). Which of the following represents the surface marking of aortic valve?

a. Sternal end of left 3rd costal cartilage

b. Sternal end of right 3rd costal cartilage

c. Beside sternum in right 2nd intercostals space

d. Besides the sternum in the right 3rd intercostals space

Solution. (a) Sternal end of left 3rd costal cartilage


Ref.:Read the text below
Sol :

Correct Answer. a

Copyright © 2014 Delhi Academy of Medical Sciences, All Rights Reserved. 8/94
(11). Triangle of Koch is the landmark for :

a. SA node

b. Bundle of His

c. AV node

d. Purkinje fibres

Solution. (c) AV node


Ref.:Read the text below
Sol :
Triangle of Koch
Triangular area in right atrium
- Marks the site of AV NODE
Boundaries
- Anterior : Base of septal leaflet of tricuspid valve
- Posterior : Anteromedial margin of opening of coronary sinus
- Superiorly : Tendon of Todaro

Correct Answer. c

(12). A 62-year-old man is incapable of penile erection after rectal surgery with prostatectomy. The patient most likely has a lesion of which of
the
following nerves?

a. Perineal nerve

b. Hypogastric nerve

c. Sacral splanchnic nerve

d. Pelvic splanchnic nerve

Solution. (d) Pelvic splanchnic nerve


Ref:Read the text below
Sol:
- The pelvic splanchnic nerve contains preganglionic parasympathetic fibers,
whereas the sacral splanchnic nerve contains preganglionic sympathetic fibers.
- Parasympathetic fibers are responsible for erection, whereas sympathetic fibers are involved with ejaculation.
- The right and left hypogastric nerves contain primarily sympathetic fibers and visceral sensory fibers.
- The dorsal nerve of the penis and the perineal nerve provide sensory nerve fibers.

Correct Answer. d

(13). A 26-year-old man presents withdecreased function in his left arm. Physical examination reveals a winged left scapula and an inability to
raise his left arm above the horizontal. Which of the following nerves is most likely affected?

a. Axillary

b. Long thoracic

c. Lower subscapular

d. Suprascapular

Solution. (b) Long thoracic


Ref:Read the text below
Sol:
- The serratus anterior, innervated by the long thoracic nerve, is responsible for stabilization of the scapula during abduction of the arm
from 90 to 180 degrees.
- When the long thoracic nerve is damaged, it is difficult to elevate the arm above the horizontal. This nerve arises from C5, 6, and 7.
- Remember: a “winged scapula” is a classic clue for long thoracic nerve injury

Correct Answer. b

Copyright © 2014 Delhi Academy of Medical Sciences, All Rights Reserved. 9/94
(14). Cooper’s ligament, is comprised of the following ligaments :

a. Inguinal and lacunar

b. Piriformis and perineal

c. Urogenital and cardinal

d. Pectineal and iliopectineal

Solution. (d) Pectineal and iliopectineal


Ref:Novak’s Gynecology 12th -74
Sol:
- The Cooper’s ligament is used frequently in the performance of a bladder neck suspension using the retropubic approach i.e., the Burch
operation.
- Cooper’s ligament is a strong ridge of fibrous tissue extending along the pectineal line (also known as the pectineal ligament), which
merges laterally with the iliopectineal ligament and medially with the lacunar ligament.

Correct Answer. d

(15). At what stage of embryonic development does an embryo normally begin to implant in the endometrium?

a. Blastocyst

b. Four-cell stage

c. Morula

d. Trilaminar embryo

Solution. (a) Blastocyst


Ref:Read the text below
Sol:
- After fertilization, the fertilized ovum begins to divide as it migrates through the uterine tube.
- It reaches the blastocyst stage (approximately 110 cells) at about day 5, and it enters the uterus at about day 6.
- Implantation normally begins on day 6 with the syncytiotrophoblast of the embryonic pole of the blastocyst eroding into the
endometrium.

Correct Answer. a

(16). For general clinical descriptions, which of the following planes is used as one of the planes to define four quadrants of the abdominal
cavity?

a. Subcostal

b. Transtubercular

c. Midclavicular

d. Transumbilical

Solution. (d) Transumbilical


Ref:Essential Clinical Anatomy by Moore- p 17.
Sol:
- The transumbilical and median planes divide the abdomen into four
quadrants
- The transverse plane (also called the horizontal plane, axial plane, or
transaxial plane) is an imaginary plane that divides the body into superior and inferior parts.
- It is perpendicular to the coronal and sagittal planes.

Correct Answer. d

Copyright © 2014 Delhi Academy of Medical Sciences, All Rights Reserved. 10/94
(17). Where is extraperitoneal fat located?

a. Between the abdominal oblique muscles

b. Deep to the parietal peritoneum

c. Superficial to camper’s fascia

d. Deep to the transversalis fascia

Solution. (d) Deep to the transversalis fascia


Ref:Essential Clinical Anatomy by Moore p 179).
Sol:
- The parietal peritoneum is internal to the transversalis fascia and is
separated from it by a variable amount of endoabdominal (extraperitoneal) fat.
- Camper’s fascia and the deep fasciae are all superficial to the extraperitoneal fat

Correct Answer. d

(18). Deep dorsal vein

a. Lies superficial to Buck's fascia

b. Drains into the prostatic venous plexus

c. Lies lateral to the dorsal artery of the penis

d. Is found in the corpus spongiosum

Solution. (b) Drains into the prostatic venous plexus


Ref:Read the text below
Sol:
- The deep dorsal vein of the penis lies medial to the dorsal artery of the penis on the dorsum of the penis and deep to Buck's fascia,
drains into the prostatic plexus of veins, and is compressed against the underlying deep fascia of the penis during erection.

Correct Answer. b

(19). Which of the following lobes of the prostate gland is commonly involved in benign hypertrophy that obstructs the prostatic urethra?

a. Anterior lobe

b. Middle lobe

c. Right lateral lobe

d. Left lateral lobe

Solution. (b) Middle lobe


Ref:Read the text below
Sol:
- The middle lobe of the prostate gland is commonly involved in benign prostatic hypertrophy, resulting in obstruction of the prostatic
urethra, whereas the posterior lobe is commonly involved in carcinomatous transformation.
- The anterior lobe contains little glandular tissue, and the two lateral lobes on either side of the urethra form the major part of the
gland.

Correct Answer. b

Copyright © 2014 Delhi Academy of Medical Sciences, All Rights Reserved. 11/94
(20). A 19-year-old woman presents with 3-cm laceration over the lateral aspect of the knee penetrating so deep that the head of the fibula is
apparent. A radiograph of the leg is negative for any fractures. A noticeable footdrop occurs while the patient walks. Furtherexamination
reveals inability to dorsiflex or evert the foot. Which of the following best explains her injury?

a. The common peroneal nerve was severed

b. The peroneal muscles were detached from the head of the fibula

c. The superficial peroneal nerve was severed

d. The tibial nerve was severed.

Solution. (a) The common peroneal nerve was severed


Ref:Read the text below
Sol:
- The common peroneal nerve wraps around the lateral aspect of the head of the fibula and is highly susceptible to damage during
lacerations or blunt injuries to the lateral knee.
- Foot-drop with loss of dorsiflexion or eversion is characteristic.

Correct Answer. a

(21). Which of the following nerves are intimately related to the humerus and are most likely to be injured by upper humerus fracture?

a. Axillary and musculocutaneous

b. Radial and ulnar

c. Radial and axillary

d. Median and musculocutaneous

Solution. (c) Radial and axillary


Ref:Read the text below
Sol:
- The axillary nerve passes posteriorly around the surgical neck of the humerus, and the radial nerve lies in the radial groove of the
middle of the shaft of the humerus.
- The ulnar nerve passes behind the medial epicondyle, and the median nerve is vulnerable to injury by supracondylar fracture of the
humerus, but these nerves lie close to or in contact with the lower portion of the humerus.
- The musculocutaneous nerve is not in direct contact with the humerus.

Correct Answer. c

Copyright © 2014 Delhi Academy of Medical Sciences, All Rights Reserved. 12/94
(22). A 17-year-old woman develops sudden difficulty breathing, with involvement of the structure identified in Image B. Which of the
following conditions is most likely to cause this patient's sudden problem?

a. Blunt trauma to the neck

b. Infection with Candida albicans

c. Aspiration of a foreign body

d. Chronic untreated hypertension

Solution. (c) Aspiration of a foreign body


Ref:Read the text below
Sol :
- The trachea is a large airway that is unlikely to be obstructed by infection, and it is in a central location not likely to be traumatized.
- However, an inhaled foreign body such as a balloon or coin, can obstruct this airway.

Correct Answer. c

(23). Medial branch of the external carotid artery is:

a. Ascending pharyngeal

b. Posterior auricular artery

c. Inferior thyroid artery

d. Superficial temporal artery

Solution. (a) Ascending pharyngeal


Ref: Read the text below
Sol :
The external carotid gives the following branches – Anterior
a. superior thyroid ( First anterior branch ).
b. lingual
c. facial
Posterior
a. posterior auricular
b. occipital
Medial
Ascending pharyngeal ( First branch )
Terminal
a. superficial temporal
b. maxillary
Inferior thyroid artery is a branch of subclavian.

Correct Answer. a

Copyright © 2014 Delhi Academy of Medical Sciences, All Rights Reserved. 13/94
(24). Dorsal scapular artery is a branch of:

a. Subclavian artery

b. Dorsal aorta

c. Arch of aorta

d. None

Solution. (a) Subclavian artery


Ref: Read the text below
Sol :
The subclavian artery is divided into three parts by the scaleneus anterior muscle.
Branches are as follows –
1st part – vertebral, internal thoracic & thyrocervical trunk
2nd part – costocervical trunk
3rd part – dorsal scapular.

Correct Answer. a

(25). Sciatic bed includes all the following Except:

a. Piriformis

b. Obturator internus

c. Adductor magnus

d. Superior gemellus

Solution. (a) Piriformis


Ref– Read the text below
Sol:
- Sciatic nerve comes below the piriformis.
- Thus not included in sciatic bed.
- The bed is contributed by- superior gemellus, obturatot internus, inferior gemellus, Quadratus femoris & adductor magnus.

Correct Answer. a

(26). The Left superior intercostal vein drains into :

a. Azygous vein

b. Right brachiocephalic vein

c. Left brachiocephalic vein

d. Hemiazygous vein

Solution. (c) Left brachiocephalic vein


Ref– Read the text below
Sol:
- The 1st left posterior intercostal vein drains into the left brachiocephalic vein.
- The 2nd , 3rd , & 4th left posterior intercostal veins join to form the left superior
intercostals vein that drains into the left brachiocephalic vein.
- The 5th , 6th, 7th& 8th join to form the accessory hemiazygous vein which drain in the azygous vein
- The 9th, 10th , 11th& 12th join to form the hemiazygous vein which also drain into the azygous vein.

Correct Answer. c

Copyright © 2014 Delhi Academy of Medical Sciences, All Rights Reserved. 14/94
(27). The vein which gets dialated in obstruction to the Superior vena cava above the opening of azygous vein is:

a. Lateral thoracic

b. Superior epigastric

c. Superficial epigastric

d. Thoracoepigastric

Solution. (d) Thoracoepigastric


Ref– Read the text below
Sol:
-The thoraco-epigastric vein connects lateral thoracic vein( tributary of Axillary ) with the superficial epigastric ( tributary of Great
Saphaneous vein ).
- All these three veins getsdialated in cases of obstruction to the SVC below the opening of azygous vein.

Correct Answer. d

(28). Eample of a typical intercostals nerve is :

a. 1st

b. 2nd

c. 3rd

d. 7th

Solution. (c) 3rd


Ref– Read the text below
Sol:
- A typical intercostal nerve is the one which supplies only the muscles of
intercostal space. These are T3 – T6.
- The first intercostals nerve (T1 ) joins the brachial plexus to supply the upper limb.
- The second intercostals nerve ( intercosto-brachial nerve) joins with the
medial cutaneous nerve of arm, to supply the skin on medial aspect of arm.
- The 7th – 12th intercostal nerves supply the muscles of anterior abdominal wall. Hence atypical.

Correct Answer. c

(29). The structures passing through optic canal include all of the following except:

a. Maxillary artery

b. Ophthalamic artery

c. Optic nerve

d. Central vein of retina

Solution. (a) Maxillary artery


Ref: Read the text below
Sol :
- The optic nerve which contains the central artery & vein of retina & the ophthalmic artery pass through optic canal.
- The maxillary artery is a branch of external carotid & is divided into three parts by lateral pterygoid muscle. It lies in the infratemporal
fossa.
- The maxillary nerve passes through foramen rotundum.

Correct Answer. a

Copyright © 2014 Delhi Academy of Medical Sciences, All Rights Reserved. 15/94
(30). The foramen ovale transmits all except:

a. Maxillary nerve

b. Mandibular nerve

c. Lesser petrosal nerve

d. Accessory menengial artery

Solution. (a) Maxillary nerve


Ref: Read the text below
Sol :
- The maxillary nerve leaves cranium through foramen rotundum, enters the pterygopalatine fossa & emerges below the orbit through
infraorbital
foramen as the infraorbital nerve.
- Mandibular nerve &lesser petrosal nerve & accessory menengial artery leave the skull through foramen ovale.

Correct Answer. a

(31). Central sulcus is an example of :

a. Limiting sulcus

b. Complete sulcus

c. Axial sulcus

d. None of the above

Solution. (a) Limiting sulcus


Ref: Read the text below
Sol :
- Central sulcus is an example of Limiting sulcus .
- It separates the motor from sensory areas.
- Calcarine & Collateral sulcus are examples of Complete sulcus.
- The calcarine sulcus is also an eg of Axial sulcus ( grows in the axis of visual
lobe ).

Correct Answer. a

(32). Callosal sulcus contains one of the following :

a. Deep middle cerebral vein

b. Middle cerebral artery

c. Perficial middle cerebral vein

d. Anterior cerebral artery

Solution. (d) Anterior cerebral artery


Ref: Read the text below
Sol :
- The callosal sulcus contains the anterior cerebral artery.
- Middle cerebral artery is present in the lateral sulcus.

Correct Answer. d

Copyright © 2014 Delhi Academy of Medical Sciences, All Rights Reserved. 16/94
(33). The aponeuroses of all three flat muscles of the anterolateral abdominal wall interlace in which of the following structures?

a. Inguinal ligament

b. Transversalis fascia

c. Linea alba

d. Anterior superior iliac spine

Solution. (c) Linea alba


Ref:Essential Clinical Anatomy by Moore p 180)
Sol:
- The aponeuroses of these muscles interlace at the linea alba with their
fellows of the opposite side to form the tough, aponeurotic tendinous sheath of the rectus muscle, and the rectus sheath

Correct Answer. c

Copyright © 2014 Delhi Academy of Medical Sciences, All Rights Reserved. 17/94
(34). Which of the following attains adult size before birth?

a. Ear Ossicles

b. Maxilla

c. Mastoid

d. Parietal bone

Solution. (a) Ear Ossicles


Ref:Langman's Medical Embryology 10thed pg 130,331
Sol:
- The viscerocranium, which consists of the bones of the face, is formed
mainly from the first two pharyngeal arches.
- The first arch gives rise to a dorsal portion, the maxillary process, which
extends forward beneath the region of the eye and gives rise to the maxilla, the
zygomatic bone, and part of the temporal bone.
- The ventral portion, the mandibular process, contains the Meckel cartilage.
Mesenchyme around the Meckel cartilage condenses and ossifies by membranous
ossification to give rise to the mandible.
- The Meckel cartilage disappears except in the sphenomandibular ligament.
- The dorsal tip of the mandibular process, along with that of the second
pharyngeal arch, later gives rise to the incus, the malleus, and the stapes
- Ossification of the three ossicles begins in the fourth month, making these the
first bones to become fully ossified.
- Mesenchyme for formation of the bones of the face is derived from neural
crest cells, including the nasal and lacrimal bones
About Ear ossicles
- The malleus and incus are derived from cartilage of the first pharyngeal arch, and the stapes is derived from that of the second arch.
- Although the ossicles appear during the first half of fetal life, they remain embedded in mesenchyme until the eighth month when the
surrounding tissue dissolves.
- The endodermal epithelial lining of the primitive tympanic cavity then extends along the wall of the newly developing space.
- The tympanic cavity is now at least twice as large as before.
- When the ossicles are entirely free of surrounding mesenchyme, the endodermal epithelium connects them in a mesentery-like fashion
to the wall of the cavity. The supporting ligaments of the ossicles develop later within these mesenteries.

Correct Answer. a

(35). A 22-year-old patient has dryness of the corneal surface of his eye because of a lack of tears. Which of the following nerves may be
damaged?

a. Proximal portion of the lacrimal nerve

b. Zygomatic branch of the facial nerve

c. Lesser petrosal nerve

d. Greater petrosal nerve

Solution. (d) Greater petrosal nerve


Ref:Read the text below
Sol:
- The secretomotor fibers to the lacrimal gland are parasympathetic fibers that run in the facial, greater petrosal, vidian (nerve of the
pterygoid canal), maxillary, zygomatic (of maxillary), zygomaticotemporal, and lacrimal (terminal portion) nerves.
- The lesser petrosal nerve carries secretomotor (preganglionic parasympathetic) fibers to the parotid gland.
- The deep petrosal nerve contains postganglionic sympathetic fibers.
- The zygomatic branch of the facial nerve supplies the facial muscles.

Correct Answer. d

Copyright © 2014 Delhi Academy of Medical Sciences, All Rights Reserved. 18/94
(36). Which of the following nerves runs through foramen rotundum?

a. Ophthalmic nerve

b. Mandibular nerve

c. Maxillary nerve

d. Optic nerve

Solution. (c) Maxillary nerve


Ref:Read the text below
Sol:
- The maxillary nerve runs through the foramen rotundum; the ophthalmic nerve runs through the supraorbital fissure; the mandibular
nerve passes through the foramen ovale; the optic nerve runs through the optic canal; and the trochlear nerve passes through the
superior orbital fissure.

Correct Answer. c

(37). A 46-year-old man complains of dryness of the mouth. On examination he lack salivary secretion from the submandibular gland. This
indicates a lesion of which of the following nervous structures?

a. Lingual nerve at its origin

b. Chorda tympani in the middle ear cavity

c. Superior cervical ganglion

d. Lesser petrosal nerve

Solution. (b) Chorda tympani in the middle ear cavity


Ref:Read the text below
Sol:
- The chorda tympani nerve contains preganglionic parasympathetic fibers responsible for secretion of the submandibular gland.
- The lingual nerve at its origin is not yet joined by the chorda tympani. The superior cervical ganglion provides sympathetic fibers, which
supply blood vessels in the submandibular gland.
- The lesser petrosal nerve contains preganglionic parasympathetic fibers
that synapse in the otic ganglion.
- The auriculotemporal nerve contains postganglionic parasympathetic fibers, which are responsible for secretion of the parotid gland.

Correct Answer. b

(38). During a gang fight, a 17-year-old boy is punched, and his nasal septum is broken. Which of the following structures would be damaged?

a. Septal cartilage and nasal bone

b. Inferior concha and vomer

c. Vomer and perpendicular plate of ethmoid

d. Septal cartilage and middle concha

Solution. (c) Vomer and perpendicular plate of ethmoid


Ref:Read the text below
Sol:
- The nasal septum is formed primarily by the vomer, the perpendicular plate of ethmoid bone, and the septal cartilage.
- The superior, middle, and inferior conchae form the lateral wall of the nasal cavity. The ethmoid (cribriform plate), nasal, frontal, and
sphenoid (body) bones form the roof.
- The floor is formed by the palatine process of the maxilla and the horizontal plate of the palatine bone.

Correct Answer. c

Copyright © 2014 Delhi Academy of Medical Sciences, All Rights Reserved. 19/94
(39). Which of the following do not elevate the ribs?

a. Serratus posterior superior

b. Serratus posterior inferior

c. External intercostals

d. Levatores costarum

Solution. (b) Serratus posterior inferior


Ref:Essential Clinical Anatomy by Moore-84
Sol:
- Serratus posterior inferior depresses the ribs

Correct Answer. b

(40). Which of the following is not a branch of the right coronary artery?

a. SA nodal

b. Right marginal

c. Posterior interventricular

d. Circumflex

Solution. (d) Circumflex


Ref:Essential Clinical Anatomy by Moore-134
Sol:
- The branches of the right coronary artery include the SA nodal, right marginal, posterior interventricular, and AV nodal arteries

Correct Answer. d

(41). Which of the following is not contained in the superior mediastinum?

a. Thymus

b. Great vessels and their branches

c. Azygos and hemiazygos veins

d. Trachea

Solution. (c) Azygos and hemiazygos veins


Ref:Essential Clinical Anatomy by Moore-158
Sol:
- The superior mediastinum contains the thymus,great vessels and their branches, vagus nerves, phrenic nerves, cardiac plexus, left
recurrent laryngeal nerve, trachea, esophagus, thoracic duct, and prevertebral muscles

Correct Answer. c

Copyright © 2014 Delhi Academy of Medical Sciences, All Rights Reserved. 20/94
(42). The thymus is supplied by which of the following?

a. Anterior intercostal branches of the internal thoracic artery

b. Brachiocephalic artery

c. Posterior intercostal arteries

d. Left common carotid artery

Solution. (a) Anterior intercostal branches of the internal thoracic artery


Ref:Essential Clinical Anatomy by Moore-142
Sol:
- The thymus is supplied by the anterior intercostal and anterior mediastinal branches of the internal thoracic artery

Correct Answer. a

(43). Which of the following is not a branch of the aorta in the thorax?

a. Posterior intercostal arteries

b. Inferior phrenic artery

c. Bronchial arteries

d. Esophageal arteries

Solution. (b) Inferior phrenic artery


Ref:Essential Clinical Anatomy by Moore-145
Sol:
- Branches of the aorta in the thorax include the posterior intercostal, bronchial, esophageal, and superior phrenic arteries

Correct Answer. b

Copyright © 2014 Delhi Academy of Medical Sciences, All Rights Reserved. 21/94
(44). A 57-year-old man has decreased vision. Which of the following neoplasms is most likely to arise in the structure identified in Image C?

a. Adenocarcinoma

b. Melanoma

c. Astrocytoma

d. Non-Hodgkin's Lymphoma

Solution. (b) Melanoma


Ref:Read the text below
Sol :
- The most common intra-ocular tumor in adults is melanoma, though intraocular tumors are, overall, not common.
- Melanomas may arise in pigmented epithelium of the uvea..

Correct Answer. b

(45). The levator ani muscle is composed of several muscles that comprise that pelvic floor and support the pelvic viscera. The muscles are:

a. Piriformis obturator internus, and iliopsoas

b. Bulbocavernosus, deep and superficial perineal

c. Puboccoccygeus, iliococcygeus, and coccygeus

d. Superficial transverse perineal, ischiocavernosus, and gluteus maximus.

Solution. (c) Puboccoccygeus, iliococcygeus, and coccygeus


Ref:Novak’s Gynecology 12th – 75,120
Sol:
- The levator ani are the supporting muscular structure for the pelvic floor. The pubococcygen can also be divided into the pubovaginalis,
the puborectalis, and the iliococcygeus.
- Together they form a broad, curved sheet of muscle that stretches from the symphysis pubis anteriorly and the coccygeus posteriorly
and from one side of the pelvis to another.
- They are perforated by the urethra, vagina, and anal canal. The levator ani assists the anterior abdominal wall in containing the
abdominal and pelvic contents.
- This important group of muscles supports the posterior wall of the vagina, facilitates defecation, and is an important component of the
mechanism of fecal continence.

Correct Answer. c

Copyright © 2014 Delhi Academy of Medical Sciences, All Rights Reserved. 22/94
(46). Three weeks after road side accident 45 yrs old male has left-sided facial swelling and discoloration, slight drooping of the left eyelid,
and a
constricted pupil on the left side.Most likely caused of this patient’s ocular disorder?

a. A lesion of cranial nerve VI

b. A lesion of the midbrain

c. A lesion of the oculomotor nerve

d. A lesion of the thoracic spinal cord

Solution. (d) A lesion of the thoracic spinal cord


Ref: Read the text below
Sol:
- The papillary dilator muscle is innervated by the sympathetic nervous system. Preganglionic fibers originating at spinal cord levels TI
through T5 ascend to the superior cervical ganglion, from which postganglionic fibers travel to the eye.
- Mueller’s muscle in the upper eyelid is also innervated by sympathetic fibers originating in the upper thoracic spinal cord.
- Disruption of the sympathetic supply to the eye, as has apparently occurred in this patient, causes ptosis and miosis due to unopposed
action of the papillary constrictor muscle.
- Interruption of sympathetic supply to the eye is known as Horner syndrome.

Correct Answer. d

(47). Most common type of tracheoesophageal fistula likely results from

a. Failure of the buccopharyngeal membrane to rupture.

b. Failure of the tracheoesophageal ridges to fuse

c. Incomplete formation of the septum secundum

d. Incomplete recanalization of the larynx

Solution. (b) Failure of the tracheoesophageal ridges to fuse


Ref:Read the text below
Sol:
- The tracheoesophageal ridges are two longitudinal ridges the separate the respiratory diverticulum from the foregut.
- Eventually, they fuse to form a septum separating the esophagus (dorsal) from the trachea (ventral) and lung buds, maintaining a
communication only rostrally at the pharynx.
- Incomplete formation of the tracheoesophageal septum (by fusion of ridges) results in the most common type of tracheoesophageal
fistula, whereby the proximal part of the esophagus ends as a blind sac (esophageal atresia), while the distal part is connected to the
trachea by a narrow canal just above the bifurcation. This defect occurs in approximately 1 in 2500 births.

Correct Answer. b

(48). The somesthetic association areas of brain include:

a. 3

b. 7

c. 6

d. 17

Solution. (b) 7
Ref: Read the text below
Sol :
- The primary sensory areas are no 3,1,2.
- The sensory association areas include areas no 5 & 7.

Correct Answer. b

Copyright © 2014 Delhi Academy of Medical Sciences, All Rights Reserved. 23/94
(49). Unhappy triad is

a. Anterior cruciate and lateral collateral ligaments and the lateral meniscus

b. Anterior cruciate and medial collateral ligaments and the medial meniscus

c. Posterior cruciate and lateral collateral ligaments and the lateral meniscus

d. Posterior cruciate and medial collateral ligaments and the lateral meniscus

Solution. (b) Anterior cruciate and medial collateral ligaments and the
medial meniscus
Ref:Read the text below
Sol :
- The anterior cruciate and medial collateral ligaments and the medial meniscus are sometimes called the “unhappy triad,” because they
are commonly injured in lateral blows to the knee that forcefully abduct the tibia. A good mnemonic is “MAMM” (for Medial collateral.
Anterior cruciate, and Medial Meniscus). Damage to the anterior cruciate, and Medial Meniscus).
- Damage to the anterior cruciate ligament is characterized by the ability to push the tibia too far forward on the femur.

Correct Answer. b

(50). A mother brings her 5-year-old daughter because the child still seems to regularly wet herself despite having been toilet-trained.IV
pyelogram, which demonstrates a complex duplication of the ureter on the right side. Although one ureter opens normally into the
bladder, the other opens ectopically into the vagina. This congenital anomaly is caused by

a. Duplication of the proximal mesonephos (Wolffian) duct

b. Early division of the ureteric bud

c. Ectopic growth of the pronephros

d. Improper union of excretory tubules and collecting tubules

Solution. (b) Early division of the ureteric bud


Ref:Read the text below
Sol :
- The collecting system of the kidney, including the ureters, develops from an outgrowth of the distal mesonephric duct called the
ureteric bud. The ureteric bud grows into the metanephros and induces the formation of the excretory system (nephrons).
- Early splitting of the ureteric bud may result in partial or complete duplication of the ureter. One of the buds typically opens into the
bladder. The lower, abnormal bud usually opens more inferiorly and may communicate with the vagina or urethra in females.

Correct Answer. b

(51). After a motor vehicle accident, a patient is brought to the emergency room. X-rays reveal that she has fractures of her left ninth and
tenth ribs. She has a rapid heart rate and low blood pressure. Peritoneal lavage reveals free blood in the peritoneal cavity. A surgeon is
able to stop the bleeding by placing a clamp across which of the following structures ?

a. Broad ligament

b. Falciform ligament

c. Gastrosplenic ligament

d. Splenorenal ligament.

Solution. (d) Splenorenal ligament.


Ref:Read the text below
Sol :
 This patient has a ruptured spleen. The splenic artery and vein reach the
spleen by passing through the splenorenal ligament.
- A clamp across this ligament will stop the flow of blood to the spleen. The spleen lies under cover of the left ninth, tenth, and eleventh
ribs.
- Fractures of these ribs may cause laceration of the spleen and bleeding
into the peritoneal cavity.

Correct Answer. d

Copyright © 2014 Delhi Academy of Medical Sciences, All Rights Reserved. 24/94
(52). A resident physician is performing a pelvic examination on a young woman. The fingers of one of her hands are in the patient’s vagina,
palpating the cervix. The other hand is pressing on the abdomen. With the palm of this hand, the physician feels a bony structure in the
lower midline. This structure is most likely the

a. Coccyx

b. Ilium

c. Ischium

d. Pubis

Solution. (d) Pubis


Ref:Read the text below
Sol :
- The resident is feeling the two pubic bones, which are joined at the midline by the symphysis pubis.
- Experienced obstetrician/gynecologists can often perceive the softening of the symphysis pubis that occurs during late pregnancy as a
“springiness” of the pubic bones during palpation.
- The coccyx is the caudal terminus of the vertebral column, generally by the fusion
of four rudimentary vertebral bodies. It is also called the tailbone.

Correct Answer. d

(53). Which of the following is the likely functional manifestation of ASD during life ?

a. No cyanosis occurred prenatally or postnatally

b. Postnatal cyanosis due to a shunt of blood from the left atrium to the right atrium

c. Postnatal cyanosis due to shunt of blood from the right atrium to the left atrium

d. Prenatal cyanosis due to a shunt of blood from the right atrium to the left atrium

Solution. (a) No cyanosis occurred prenatally or postnatally


Ref:Read the text below
Sol :
- Atrial septal defect is a noncyanotic defect. This defect will result in postnatal shunting of blood from the left to the right atrium.
Because the left atrium contains oxygenated blood, this shunt results in oxygenated blood being sent back to the pulmonary circuit.
- Cyanosis is the result of deoxygenated blood being sent to the systemic circuit. Heart defects that result in postnatal shunts from right
to left are cyanotic defects because deoxygenated blood on the right side of the heart is shunted to the left side of the heart, which sends
blood into the systemic circulation.

Correct Answer. a

(54). A 6-year-old boy presents with neck mass. His mother reports that he first developed the mass 3 days ago, and it has not
resolved.Physical
examination shows a 2-cm neck mass in the midline immediately above the thyroid cartilage notch. The mass elevates when he sticks his
tongue out of his mouth. Which of the following is the most likely diagnosis ?

a. Reactive lymph node

b. Branchial cleft cyst

c. Thyroglossal duct cyst

d. Head and neck neoplasm

Solution. (c) Thyroglossal duct cyst


Ref:Read the text below
Sol :
- Thyroglossal duct cysts are derived from the remnant of the thyroglossal duct.
- The thyroglossal duct extends from the foramen cecum of the tongue base to the thyroid gland during the embryologic development of
the thyroid gland, and it typically degenerates completely.
- If a portion of the duct remains, it can develop into a cyst and enlarge,
presenting as a midline neck mass.
- The path gnomonic finding of a thyroglossal duct is elevation during
protrusion of the tongue, which is true in this case.

Correct Answer. c

Copyright © 2014 Delhi Academy of Medical Sciences, All Rights Reserved. 25/94
(55). A patient presents to the physician complaining of “something wrong with her foot” that causes her to trip and fall. Physical examination
revels an inability to dorsiflex (extend) the foot and a loss of sensation between the first and second toes. There is no other motor or
sensory loss. Which of the following nerves was most likely injured ?

a. Deep peroneal

b. Femoral

c. Superficial peroneal

d. Sural

Solution. (a) Deep peroneal


Ref:Read the text below
Sol :
- The deep peroneal nerve arises from the common peroneal nerve (L4-S2). It innervates the muscles of the anterior compartment of the
leg, which dorsiflex (extend) the foot. Damage to the nerve therefore produces “foot-drop” – a classic clue to deep peroneal nerve
pathology.
- The nerve gives rise to many branches, one of which innervates the skin
between the first and second toes.

Correct Answer. a

(56). A surgeon performing an appendectomy makes an incision through the ventrolateral abdominal wall. The layers of the abdominal wall
are (1) Internal oblique (2) External oblique (3) Peritoneum (4) Transversus abdominis Which of the following corresponds to the order of
penetration of the layers of the abdominal wall ?

a. 1-3-4-2

b. 2-1-3-4

c. 2-1-4-3

d. 1-2-3-4

Solution. (c) 2-1-4-3


Ref:Read the text below
Sol :
- Questions like these are particularly amenable to the use of test-taking
strategies because even if you don’t know 100% of the answer, you will be able to eliminate some answer choices on the basis of
knowledge you do have.
- For example, you probably are aware that the peritoneum does not lie
superficial to the transverses abdominis muscle, enabling you to eliminate
other three choices.

Correct Answer. c

(57). A woman comes to the physician for a prenatal visit. Examination reveals that her uterus is considerably larger than her gestational age
would predict. An ultrasound examination is ordered and reveals that she has polyhydramnios. Which of the following congenital defects
of the fetus would be most likely to be associated with this abnormality ?

a. Atrial septal defect

b. Esophageal atresia

c. Lung hypoplasia

d. Meckel diverticulum

Solution. (b) Esophageal atresia


Ref:Read the text below
Sol :
- Polyhydramnios is the presence of excess amniotic fluid. Because amniotic fluid is normally swallowed by the fetus, esophageal atresia,
an obstruction of the esophagus that prevents the fetus from swallowing the amniotic fluid, an excess of fluid is retained in the amniotic
cavity.
- An atrial septal defect is a common acyanotic heart defect. This defect has no relationship to abnormalities in amniotic fluid volume.

Correct Answer. b

Copyright © 2014 Delhi Academy of Medical Sciences, All Rights Reserved. 26/94
(58). A newborn baby is observed to be cyanotic immediately after birth. Diagnostic studies, including an ultrasound, reveal that the baby has
persistent truncus arteriosus. Which of the following additional defects is this baby most likely to have ?

a. Dextrocardia

b. Membranous ventricular septal defect

c. Secundum-type atrial septal defect

d. Tetralogy of Fallot

Solution. (b) Membranous ventricular septal defect


Ref:Read the text below
Sol :
- Persistent truncus arteriosus results from failure of the aorticopulmonary septum to form. Normally, the aorticopulmonary septum
divides the truncus arteriosus into the ascending aorta and the pulmonary trunk.
- The aorticopulmonary septum also contributes to the formation of the
membranous portion of the interventricular septum. When the aorticopulmonary septum does not form, the truncus arteriosus persists
rather than being divided, and the membranous interventricular septum is incomplete.
- Because neural crest cells contribute to the formation of the aorticopulmonary septum, it is believed that this defect may result from a
defect in migration of neural crest cells.

Correct Answer. b

(59). Injury to the lower division of facial nerve during surgery will most likely result in

a. Inability to furrow the brow (to frown) on the same side

b. Numbness over the angle and mental region of the jaw on the same side

c. Weakness in losing the eye on the same side

d. Weakness of the lower lip on the same side

Solution. (d) Weakness of the lower lip on the same side


Ref:Read the text below
Sol :
- The motor component (special visceral efferent) of the facial nerve (CN VII) exits the skull via the stylomastoid foramen, passes lateral
to the styloid process, and then enters the parotid gland. Within the gland, two divisions can usually be identified (upper and lower),
which in turn give off five named branches that innervate the muscles of the face.
- The upper division gives rise to the temporal and zygomatic branches,
which collectively innervate the frontalis, corrugators, and orbicularis
oculi muscles.
- The lower division gives off the buccal, mandibular, and cervical branches.
 The largest, the buccal, innervates the muscles attaching to the upper lip,
including the orbicularis oris and the levators, as well as the buccinators
and the muscles of the nose.

Correct Answer. d

(60). After an amniocentesis, a 29-year-old woman who is carrying twins learns that one fetus has XY chromosomes and the other has XX
chromosomes. At 37 weeks’ gestation, the woman deliver two healthy babies. Which of the following structures is likely to be present in
the baby with the XY chromosomes but not in the one with XX chromosomes ?

a. Bulbospongiosus muscle

b. Bulbourethral gland

c. Corpus cavernosum

d. Membranous urethera

Solution. -NA-

Correct Answer. b

Copyright © 2014 Delhi Academy of Medical Sciences, All Rights Reserved. 27/94
(61). A-32-year-old man complains of progressive, severe, generalized headaches that began 3 months ago, are worse in the mornings, and
lately have been accompanied by projectile vomiting. He has also lost his upper gaze, and on physical examination the upper part of the
sclera is visible above the downward looking, irises. Which of the following is the most likely diagnosis ?

a. Nelson syndrome

b. Pituitary apoplexy

c. Posterior fossa tumor

d. Tumor in the area of the pineal gland

Solution. (d) Tumor in the area of the pineal gland


Ref:Read the text below
Sol :
- The first lines of the question clearly suggest a brain tumor. The additional ocular findings (loss of upper gaze and “sunset eyes” – the
upper part of the sclera is visible above the downward-looking irises) locate it to the pineal gland area.
- The frontal lobe when pressed on by tumors, produces changes in social
behavior and olfactory and visual deficits.
- Nelson syndrome occurs in people who have had bilateral adrenalectomies performed for Cushing disease secondary to a nonresected
pituitary microadenoma. The symptom is that of pressure on the chiasma (hemianopsia)

Correct Answer. d

(62). Arcuate fasciculus connects :

a. Temporal & frontal lobes

b. Frontal & occipital lobes

c. Temporal & parietal lobes

d. Frontal & parietal lobes

Solution. (a) Temporal & frontal lobes


Ref: Read the text below
Sol :
- White matter of CNS are classified as Association, Projection & Commissural fibres. Uncinate fasciculus is an example of association
fibers.
- They connect frontal ( Brocas area no – 44, 45 ) & temporal ( Wernikes area no 22 ) lobes.

Correct Answer. a

(63). Tract present in the middle cerebellar peduncle is

a. Ant spinocerebellar

b. Pontocerebellar

c. Olivocerebellar

d. Reticulocerebellar

Solution. (b) Pontocerebellar


Ref: Read the text below
Sol :
- The olivocerebellar fibres passes through the inferior cerebellar peduncle, pontocerebellar through MCP.
- The anterior spinocerebellar & reticulocerebellar fibres pass theough SCP.

Correct Answer. b

Copyright © 2014 Delhi Academy of Medical Sciences, All Rights Reserved. 28/94
(64). The artery which accompanies the great cardiac vein is:

a. Right coronary artery

b. Left coronary artery

c. Posterior interventricular artery

d. Anterior interventricular artery

Solution. (d) Anterior interventricular artery


Ref.:Read the text below
Sol :
Veins of the Heart
- Coronary sinus
- Largest vein of the heart situated in the left posterior coronary sulcus
- Ends by opening into the posterior wall of right atrium between IVC opening & AV orifice
- Coronary sinus opening is guarded by the besian valve (or semilunar valve of coronary sinus)

Correct Answer. d

(65). The superficial part of the cardiac plexus lies beneath the arch of the aorta, in front of the right pulmonary artery. Which of the following
helps in forming superficial cardiac plexus?

a. Inferior cervical cardiac branch of right vagus

b. Superior cervical cardiac branch of left vagus

c. Superior cervical cardiac branch of left sympathetic chain

d. Inferior cervical cardiac branch of left sympathetic chain

Solution. (c) Superior cervical cardiac branch of left sympathetic chain


Ref.:Read the text below
Sol :
Nerve supply of the Heart
Parasympathetic
- Reach heart via vagus
- Cardio inhibitory
- Afferent fibres sense alteration in BP & blood chemistry & therefore concerned with
cardiac reflex
Sympathetic :
- T2 to T5 segments of spinal cord
- Cardio acceleratory & dilate coronary arteries
- Carry pain impulses to spinal cord via T1 to T5 nerves
- Thus pain may be referred to area of skin supplied by these nerves.

Correct Answer. c

Copyright © 2014 Delhi Academy of Medical Sciences, All Rights Reserved. 29/94
(66). True about right ventricle are all except :

a. Situated posteriorly

b. Crista supraventricularis separate tricuspid & pulmonary valve

c. Outflow part is called trabeculae carneae

d. Moderator band contains right branch of AV bundle

Solution. (c) Outflow part is called trabeculae carneae


Ref.:Read the text below
Sol :
Right Ventricle
- External features
- If forms the anterior (sternocostal) & inferior (diaphragmatic) surface of heart
- AV groove lodges the right coronary artery
Internal features
- The interior has two parts
- The inlet part is rough due to muscle ridges called trabeculae carneae.
- It develops from proximal part of bulbus cordis
- The smooth outflow part is called infundibulum. It develops from mid portion of the
bulbus cordis.
- The two parts are separated by a muscular ridge called supraventricular crest or
infundibular crest between tricuspid & pulmonary orifices
Muscle rides or trabeculae carneae is of three types
- Ridges or fixed elevation
- Bridges
- Pillars or papillary muscle: The 3 papillary muscle are anterior (largest), posterior or
inferior (small & irregular) and septal (divided into number of little nipples)
The septo marginal trabeculae or moderator band is a muscular ridge
extending from ventricular septum to the base of the anterior papillary
muscle. It contains right branch of AV bundle.
- The cavity is crescentic in cross section.
- The wall is thinner than LV in a ratio of 1:3

Correct Answer. c

(67). The fibrous tissue between aortic & mitral rings is called:

a. Trigonum fibrosum dextrum

b. Trigonum fibrosum sinistrum

c. Tendon of infundibulum

d. Crista terminalis

Solution. (b) Trigonum fibrosum sinistrum


Ref.:Read the text below
Sol :
Atrioventricular fibrous rings
- It is arranged in the form of figure of ‘8’
- Atria, ventricles and membranous part of interventricular septum are attached to it
- This ring acts as a barrier between atria and ventricle so that electrical impulses can be transmitted from atria to ventricle only via AV
bundle
Trigonum fibrosum dextrum
- It is a large mass of fibrous tissue between the atrioventricular rings behind & aortic ring in front Trigonum fibrosum sinistrum
- It is present between aortic & mitral rings
Conus ligament (tendon of infundibulum)
- It connects pulmonary infundibulum posteriorly to the root of aorta

Correct Answer. b

Copyright © 2014 Delhi Academy of Medical Sciences, All Rights Reserved. 30/94
(68). The apex of the lung extends into the root of the neck, reaching between 2.5 cm and 4 cm above the level of the sterna end of the first
rib.
All of the following lie anterior to the apex of lung except:

a. Thoracic duct

b. Subclavian artery

c. Internal thoracic artery

d. Scalenus anterior

Solution. (a) Thoracic duct


Ref.:Read the text below
Sol :
Relations of lung apex
In front (Anterior)
Subclavian artery & vein
- Origin of internal thoracic artery from 1st part of subclavian artery
Scalenus anterior muscle
- Behind (Posterior)
- Apex is in contact with the 1st rib and the following structures intervene between
them (from medial to lateral) :
Sympathetic trunk
- 1st posterior intercostals artery
- Ascending branch of the ventral ramus of 1st thoracic nerve. It joins with C8 to form
the lower trunk of brachial plexus
Laterally
- Scalene medius muscle
- Lower trunk of brachial plexus

Correct Answer. a

(69). The thymus enlarges from about the 12th week of gestation until puberty, when it begins to shrink. Thymus lies in which part of the
mediastinum?

a. Superior

b. Anterior

c. Middle

d. Posterior

Solution. (b) Anterior


Ref.:Read the text below
Sol :
Anterior mediastinum
Bounded:
- Laterally by the pleurae
- Posteriorly by the pericardium
- Anteriorly by the sternum, the left transversus thoracis and the fifth, sixth, and seventh left costal cartilages
Contents
- A quantity of loose areolar tissue
- Some lymphatic vessels which ascend from the convex surface of the liver
- Two or three anterior mediastinal lymph nodes
- The small mediastinal branches of the internal thoracic artery
- Thymus (involuted in adults)

Correct Answer. b

Copyright © 2014 Delhi Academy of Medical Sciences, All Rights Reserved. 31/94
(70). The clavipectoral fascia is pierced by which of the following structures?

a. Medial pectoral nerve

b. Thoracoacromial artery

c. Subclavian vein

d. Transverse cervical artery

Solution. (b) Thoracoacromial artery


Ref.:Read the text below
Sol :
Clavipectoral Fascia
- Situated deep to clavicular portion of pectoralis major
- It splits & encloses subclavius muscle in upper part & pectoralis minor in the lower
part
Attachment :
- Upper : Clavicle & investing layer of deep cervical fascia
- Lower : Dome of axillary fascia
- Medial : 1st rib & costoclavicular ligament
- Lateral : Coracoid process & coracoclavicular ligament
Structures Piercing it :
- Lateral pectoral nerve
- Cephalic vein
- Thoracoacromial vessels
- Lymphatics passing from breast & pectoral region to the apical group of axillary lymph nodes.

Correct Answer. b

(71). 8th to 11th posterior intercostal veins of left side drains into:

a. Azygos vein

b. Hemiazygos vein

c. Accessory hemiazygos vein

d. Left brachiocephalic vein

Solution. (b) Hemiazygos vein


Ref.:Read the text below
Sol :
HEMIAZYGOS VEIN :
- Formed by union of left subcostal & left ascending lumbar veins.
- Enters thorax by piercing the left crus of diaphragm.
- Drains into azygos vein opposite T8 vertebra.
- It may communicate with the left renal vein.
- Receives blood from lower 3rd to 4th posterior intercostals veins.

Correct Answer. b

(72). Regarding embryologic development, false statement is:

a. Foramina venereum is arising from the pulmonary vessels

b. Foramina venereum is arising from the interauricular septum

c. Foramina venereum is arising from the septum

d. Orifices of venae cordis maximae

Solution. (c) Foramina venereum is arising from the septum


Ref: Read the text below
Sol :
- The foramina venarum minimarum are the orifices of venae cordis minimae which return a small fraction of blood from the heart. They
are most numerous on the septal wall.
- The anterior cardiac veins and sometimes the right marginal may enter the atrium.

Correct Answer. c

Copyright © 2014 Delhi Academy of Medical Sciences, All Rights Reserved. 32/94
(73). The origin of the medial epiphyseal centre is :

a. 3 years

b. 7 years

c. 11 years

d. 5 years

Solution. (b) 7 years


Ref: Read the text below
Sol :

Correct Answer. b

(74). Which of the following statements are true regarding root of right lung?

a. The right phrenic nerve passes anterior to the lung root

b. The azygos vein arches forward over the superior margin of the lung root.

c. The right pulmonary artery lies anterior to the principal bronchus in the lung root.

d. The right vagus nerve passes posterior to the lung root

Solution. (d) The right Vagus nerve passes posterior to the lung root
Ref: Read the text below
Sol :
ROOT OF RIGHT LUNG
A little above the middle of the mediastinal surface of each lung, and nearer its
posterior than its anterior border, is its root, by which the lung is connected to the
heart and the trachea.
The root is formed by the bronchus, the pulmonary artery, the pulmonary veins, the
bronchial arteries and veins, the pulmonary plexuses of nerves, lymphatic vessels,
bronchial lymph glands, and areolar tissue, all of which are enclosed by a reflection
of the pleura.
The root of the right lung lies behind the superior vena cava and part of the right
atrium, and below the azygos vein. That of the left lung passes beneath the aortic
arch and in front of the descending aorta; the phreni pericardiacophrenic artery and
vein, and the anterior pulmonary plexus, lie in front of each, and the vagus and posterior pulmonary plexus behind each; below each is
the pulmonary ligament.
The chief structures composing the root of each lung are arranged in a similar manner from before backward on both sides, viz., the
upper of the two pulmonary
veins in front; the pulmonary artery in the middle; and the bronchus, together with
the bronchial vessels, behind.
From above downward, on the two sides, their arrangement differs, thus:
On the right side their position is—eparterial bronchus, pulmonary artery,
hyparterial bronchus, pulmonary veins, but on the left side their position is—
pulmonary artery, bronchus, pulmonary veins.
The lower of the two pulmonary veins, is situated below the bronchus, at the apex or lowest part of the hilus.

Correct Answer. d

Copyright © 2014 Delhi Academy of Medical Sciences, All Rights Reserved. 33/94
(75). Fecal fistula at umbilicus is due to:

a. Persistent vitellointestinal duct

b. Persistent patent urachus

c. Persistent remnant of urachus

d. Ligamentum arteriosum

Solution. (a) Persistent vitellointestinal duct


Ref: Read the text below
Sol :
CAUSES OF UMBILICAL FECAL FISTULA
Congenital :
- Persistent Vitellointestinal duct: This is an uncommon anomaly. If the fistula is small it leads to mucoid discharge, but if it is large then
it results in ‘omphaloenteric fecal fistula’. Acquired:
- Internal Hollow Viscus Malignancies : As in CA: Transverse colon can erode the umbilicus leading to fecal fistula.
- TB peritonitis: A perforation (induces by TB) which is sealed off by coils of matted bowel and omentum results in local abscess which
may perforate through a weakpoint i.e. umbilicus resulting in a fecal fistula.

Correct Answer. a

(76). The proper distribution of body weight in vertebral column is via:

a. Laminae

b. Pedicle

c. Vertebral body

d. Neural arch

Solution. (c) Vertebral body


Ref: Read the text below
Sol :
There are three main parts to vertebra
- Vertebral Body (weight bearing structure)
- Vertebral Arch (protective in function)
- Vertebral Processes
- The vertebral body is a thick, disc shaped cylindrical block of bone flattened at the back and with roughened top and bottom surfaces.
- It is made up of spongy bone on the inside, which enables it to resist compression, and a thin outer covering of compact bone. It is the
weight bearing part of a vertebrae.

Correct Answer. c

(77). Posterior boundary of foramen of winslow is :

a. Aorta

b. Inferior vena cava

c. Portal vein

d. Liver

Solution. (b) Inferior vena cava


Ref: Read the text below
Sol :
- The other name for epiploic foramen is foramen of winslow.
- It is situated behind the free margin of the lesser omentum at the level of
t12 vertebra.
Relationship:
- Anteriorly: Right free margin of the lesser omentum containing portal vein, hepatic artery and bile duct.
- Posteriorly: The inferior vena cava, right suprarenal gland and t12 vertebra.
- Superiorly: The caudate process of liver.
- Inferiorly: 1st part of duodenum and horizontal part of hepatic artery.

Correct Answer. b

Copyright © 2014 Delhi Academy of Medical Sciences, All Rights Reserved. 34/94
(78). A newborn girl is noted to have abnormal accommodation of the lens. Further evaluation reveals an abnormal production of aqueous
humor. A malformation of the structure responsible for these functions that is continuous posteriorly with the choroid might be due to
abnormal development of

a. Choroid fissure of the optic stalk

b. Mesoderm surrounding the optic up

c. Mesoderm surrounding the optic stalk

d. Neuroectoderm of the optic cup

Solution. (d) Neuroectoderm of the optic cup


Ref: Read the text below
Sol:
- The optic cups derive from the optic vesicles, which are evaginations of
the dien, cephalon.
- The anterior two layers of the optic cup (neurectoderm), in association with choroidal mesoderm, give rise to the ciliarybody and the
iris.
- The optic cup also gives rise to the neural and pigment layers of the
retina.

Correct Answer. d

(79). A 70-year-old woman has had a severe headache for 3 days. A lumbar puncture is performed and microscopic examination of the CSF
shows 550 neutrophils/microliter. A gram stain of the CSF reveals gram positive clusters of cocci. Which of the following conditions in
the structure identified in Image D is most likely to cause these findings?

a. Bell's palsy

b. Giant cell arteritis

c. Eosinophilic granuloma

d. Mastoiditis

Solution. (d) Mastoiditis


Ref:Read the text below
Sol :
- The mastoid air cells can become infected via extension from a middle ear infection, and the infection can then spread to the meninges
to produce a meningitis.
- The findings in this case are typical for a Staphylococcus aureus meningitis. With antibiotic therapy of ear infections, mastoiditis is
uncommon.

Correct Answer. d

Copyright © 2014 Delhi Academy of Medical Sciences, All Rights Reserved. 35/94
(80). Which of the following nuclei is the most important source of noradrenergic innervations to the cerebral cortex?

a. Basal nucleus of Meynert

b. Caudate nucleus

c. Locus caeruleus

d. Raphe nucleus

Solution. (c) Locus caeruleus


Ref: Read the text below
Sol:
- The locus careuleus is a dense collection of neuromelanin-containing cells in the rostral pons, near the lateral edge of the floor of the
fourth ventricle.
- The fact that it appears blue-black in unstained brain tissue gave rise to its name, which means “blue sport” in Latin.
- These cells, which contain norepinephrine, provide the majority of noradrenergic innervations to the forebrain, including the cerebral
cortex.

Correct Answer. c

(81). To control the bleeding from carotid artery, the surgeon can compress the carotid artery against the anterior tubercle of which of the
following vertebrae?

a. Second cervical

b. Third cervical

c. Fourth cervical

d. Sixth cervical

Solution. (d) Sixth cervical


Ref: Read the text below
Sol:
- The sixth cervical vertebra is a critical boundary of the root of the neck. To enter the neck from the chest, the vascular structures pass
through a ringlike opening bounded by the scalene muscles laterally, the sternum and first ribs interiorly and the vertebrae (notably C6).

Correct Answer. d

(82). Which of the following structures form the lateral and medial borders of the femoral triangle?

a. Adductor longus and gracilis

b. Adductor longus and Sartorius

c. Gracilis and pectineus

d. Gracilis and Sartorius

Solution. (b) Adductor longus and Sartorius


Ref: Read the text below
Sol:
- You should know the boundaries of the femoral triangle: the inguinal ligament above, the medial border of the Sartorius laterally, and
the medial border the adductor longus on the inner aspect of the thigh.

Correct Answer. b

Copyright © 2014 Delhi Academy of Medical Sciences, All Rights Reserved. 36/94
(83). The first bony structure that can be palpated below the inferior margin of the medial protion of the clavicle is the

a. Atlas

b. First rib

c. Manubrium

d. Second rib

Solution. (d) Second rib


Ref: Read the text below
Sol:
- The palpable space immediately inferior to the clavicle is the first intercostals space, and the bone below it is the second rib.

Correct Answer. d

(84). An oblique x-ray view of a patient with right middle lobar pneumonia demonstrates an area of consolidation bounded by sharp,
interscting
relatively straight lines above and below. These lines correspond to which of the following?

a. Diaphragm above and oblique fissure below

b. Oblique fissure above and breast shadow below

c. Oblique fissure above and transverse fissure below

d. Transverse fissure above and oblique fissure below

Solution. (d) Transverse fissure above and oblique fissure below


Ref: Read the text below
Sol:
- The transverse fissure above (between the right upper and right middle lobes) and the oblique fissure below (between the right middle
and right lower lobes) are often strikingly visible on the opblique view in patients with right middle lobar pneumonia.
- The diaphragm and the breast may also be seen but do not overlap the middle lobe on the oblique view.

Correct Answer. d

(85). Tonsils develop from which of the following pharyngeal pouches?

a. First

b. Second

c. Third

d. Fourth

Solution. (b) Second


Ref: Read the text below
Sol:
- The epithelial lining of the second pharyngeal pouch bus into the mesencyme to form the palatine tonsil.
- Part of the pouch remains in the adult as the tonsillar fossa.

Correct Answer. b

Copyright © 2014 Delhi Academy of Medical Sciences, All Rights Reserved. 37/94
(86). A 35-year-old woman with severe dysmenorrheal and prolonged menstrual periods due to large uterine fibroids undergoes a
hysterectomy.
Which of the following structures is the gynecologist most likely to inadvertently ligate during surgery?

a. Internal iliac artery

b. Internal iliac vein

c. Ovarian artery

d. Ureter

Solution. (d) Ureter


Ref: Read the text below
Sol:
- The ureter passes directly inferior to the uterine artery, lateral to the body of the uterus near its junction with the cervix (“water flows
under the bridge”).
- During a hysterectomy, therefore, the ureter (instead of the uterine artery) may be inadvertently ligated.

Correct Answer. d

(87). Which of the following embryonic structures gives rise to the prostate?

a. Genital tubercle

b. Processus vaginalis

c. Testis cords

d. Urogenital sinus

Solution. (d) Urogenital sinus


Ref: Read the text below
Sol:
- The prostate is immediately derived from the prostatic urethra, which is
derived from the urgenital sinus.

Correct Answer. d

(88). A football player experiences an anterior dislocation of the shoulder. Cutaneous sensation over the lower half of the deltoid muscle is
impaired. These findings suggest damage to which of the following nerves?

a. Axillary

b. Median

c. Musculocutaneous

d. Radial

Solution. (a) Axillary


Ref: Read the text below
Sol:
- The axillary nerve can be damaged during anterior dislocation of the shoulder, causing loss of sensation in the skin overlying the lower
half of the deltoid
muscle.

Correct Answer. a

Copyright © 2014 Delhi Academy of Medical Sciences, All Rights Reserved. 38/94
(89). Duodenum is demarcated from jejunum by :

a. Ligament of treitz

b. Superior mesenteric artery

c. Falceiform ligament

d. Tail of pancreas

Solution. (a) Ligament of treitz


Ref: Read the text below
Sol :
- The duodenum is an immobile C-shaped segment of small bowel that lies just downstream to the pylorus (the opening, surrounded by
muscular tissue, from the stomach into the duodenum).
- It is secured to the region of the stomach by Treitz’s ligament, which is the dividing point between the duodenum and the jejunum.
- The duodenum lies in close proximity to the stomach, pancreas, liver,
gallbladder, and transverse colon.

Correct Answer. a

Copyright © 2014 Delhi Academy of Medical Sciences, All Rights Reserved. 39/94
(90). All of the following are correct about spinal cord except –

Copyright © 2014 Delhi Academy of Medical Sciences, All Rights Reserved. 40/94
a. The efferent nerve fibres originate from the anterior horn cells

b. The central canal is situated in the white commisure

Copyright © 2014 Delhi Academy of Medical Sciences, All Rights Reserved. 41/94
c. It has grey matter core with white matter covering

Copyright © 2014 Delhi Academy of Medical Sciences, All Rights Reserved. 42/94
d. The ligamentum denticulatum anchors the lateral borders of the spinal cord to the meninges.

Copyright © 2014 Delhi Academy of Medical Sciences, All Rights Reserved. 43/94
Solution. (b) The central canal is situated in the white commisure
Ref: Spinal Cord Medicine: Principles and Practice. Lin VW, Cardenas DD, Cutter NC,
et al
Neuroanatomic organisation of spinal cord-
Sol :
- On cross section, the spinal cord is composed of a central portion of
butterfly-shaped gray matter and peripherally oriented white matter.
- The gray matter is comprised predominantly of neurons, their processes, and glial cells and has an enriched blood supply.
- The white matter contains ascending and descending fiber tracts and glial cells and appears white in unfixed tissue because of a
predominance of myelin.
- The two halves of the gray matter are connected across the midline by
a dorsal and ventral gray commissure, which is located above and below the central canal respectively.
- The gray matter is further subdivided into a posterior (dorsal) horn (column) and an anterior (ventral) horn (column).
- The thoracic and upper two lumbar spinal cord segments also display a wedgeshaped, intermediate lateral horn (intermediolateral cell
column).
CENTRAL CANAL
- It is the conduit that runs the entire length of the spinal cord and contains some of the 140 mL of cerebrospinal fluid (CSF) in the body
of the average individual.
- The central canal of the spinal cord lies in the center of the cord between the ventral and the dorsal gray commissures and extends
toward the cranium into the medulla oblongata, where it opens into the fourth ventricle of the brain.
- Lumbar puncture, often performed to obtain samples of CSF for diagnostic purposes, draws fluid from the subarachnoid space around
the spinal cord and not from the central canal.
 The anterior horn of the spinal cord is the ventral (front)grey matter section of the spinal cord. The anterior horn contains motor
neurons also known as efferent neuronsthat affect the axial muscles while the posterior horn receives information regarding touch and
sensation. The anterior horn is where the cell bodies of alpha motor neurons are located.
- The Ligamentum Denticulatum is a narrow, fibrous band, situated on each side of the spinal cord, throughout its entire length, and
separating the anterior from the posterior roots of the spinal nerves, having received its name from the serrated appearance which it
presents.
- Its inner border is continuous with the pia mater, at the side of the cord.
- Its outer border presents a series of triangular, dentated serrations, the
points of which are fixed, at intervals, to the dura mater, serving to unite
together the two layers of the arachnoid membrane.
- These serrations are about twenty in number, on each side, the first being attached to the dura mater, opposite the margin of the
foramen magnum, between the vertebral artery and the hypoglossal nerve; and the last corresponds to nearly the lower end of the cord.
- Its use is to support the cord in the fluid by which it is surrounded.

Copyright © 2014 Delhi Academy of Medical Sciences, All Rights Reserved. 44/94
Correct Answer. b

(91). The supraduodenal bile duct is chiefly supplied by –

a. Vessels that run upwards from gastroduodenal and retroduodenal arteries

b. Vessels that run downwards from the cystic artery

c. Vessels that arise from the hepatic artery as it courses up along the common bile duct and supplies it in a non-axial fashion

d. Vessels that run downwards from the right hepatic artery

Solution. (a) Vessels that run upwards from gastroduodenal and


retroduodenal arteries
Ref: Liver: A Complete Book on Hepato-Pancreato-Biliary Diseases,
Chapter 64
Sol :
- The bile duct is sub classified into three segments hilar, supraduodenal & retopancreatic.
- The blood supply of supra-duodenal duct is essentially axial.
- It arises from the superior pancreaticoduodenal artery, right branch of hepatic artery, cystic artery, gastroduodenal artery and
retroduodenal artery.
- The most important of these vessels run along the lateral border of ducts and are called 3 ‘o’ clock & 9 ‘o’ clock arteries.
- The hilar duct receives blood from surrounding vessel forming a rich network on surface of the ducts in continuity with the plexus
around supraduodenal duct.
- The retropancreatic common bile duct gets its blood supply from retroduodenal artery.

Correct Answer. a

(92). Development of external ear is from :

a. 1st and 2nd pharyngeal arch

b. 1st and 2nd branchial cleft

c. 1st branchial cleft

d. 2nd branchial arch

Solution. (a) 1st and 2nd pharyngeal arch


Ref: Read the text below
Sol :
- Branchial Arches begin to develop early in the fourth week and appear as round ridges on each side of the future head and neck.
- By the end of the fourth week, 4 well defined pairs are visible; the 5th and 6th are rudimentary. The arches are separated from one
another by the branchial clefts or grooves. Development of the External Ear
- The auricle develops from 3 pairs of auricular hillocks that arise in the 5th week, on the lateral faces of the 1st and 2nd pharyngeal arches.
- The hillocks are named for the portions of the auricles to which they give rise.

Correct Answer. a

(93). Which muscle pair rotates radius without flexion and extension at arm?

a. Pronator quadrates and pronator teres

b. Supinator and anconeus

c. Brachioradialis and brachialis

d. Triceps and biceps brachii

Solution. (a) Pronator quadrates and pronator teres


Ref: Read the text below
Sol :
- The other word for rotation of radius is “PRONATION”
- Pronator teres is the main pronator of the forearm. It also flexes the elbow.
- The superficial fibers of pronator quadrates pronate the forearm while the deep fibers bind the lower ends of radius and ulna.

Correct Answer. a

Copyright © 2014 Delhi Academy of Medical Sciences, All Rights Reserved. 45/94
(94). Situation of motor speech is at:

a. Superior temporal gyrus

b. Inferior frontal gyrus

c. Superior frontal gyrus

d. Inferior parietal gyrus

Solution. (b) Inferior frontal gyrus


Ref: Read the text below
Sol :
- The motor speech area of broca lies in the inferior frontal gyrus
- Injury to this region results in inability (aphasia) to speak even though the muscles concerned are not paralyzed.

Correct Answer. b

(95). In dislocation of 5th and 6th cervical vertebra which of the following spinal nerve is injured?

a. VI

b. V

c. IV

d. III

Solution. (a) VI
Ref: Read the text below
Sol :
- The cervical nerve present between C5 and C6 vertebra in the 6th nerve. Hence this is the nerve that gets injured in case of dislocation
of C5 and C6.
- This injury commonly leads to paraplegia.
- Dislocation of C3 or above vertebra leads to the death of the victim.

Correct Answer. a

(96). The axial skelecton develops from :

a. Endoderm

b. Lateral plate mesoderm

c. Intermediate part of somites (i.e. primitive segments)

d. Primitive segments derived from paraxial mesoderm

Solution. (d) Primitive segments derived from paraxial mesoderm


Ref: Read the text below
Sol :
During the 3rd week the paraxial mesoderm forms into "balls" of mesoderm paired either side of the neural groove, called somites.
Different regions of the somite differentiate into dermomyotome (dermal and muscle component) and sclerotome (forms vertebral
column). Vertebral bone is formed through a lengthy process involving endochondrial ossification of a cartilage formed from
mesenchyme.
The vertebral body begins as a bony collar that expands into regions of dying cartilage. The bony vertebral arch, enclosing the spinal
cord, forms later and the arch remains open dorsally (linked by a ligament) to allow growth of the spinal cord. The axial skeleton consists
of: Skull, Auditory Ossicles, Hyoid bone, Vertebral column, Chest (sternum, ribs)
The axial skelecton develops from primitive segments derived from paraxial mesoderm The vertebral column is a series of bone segments
(vertebra) separated by specialized joints (intervertebral disc). In the adult, the vertebra form rostro-caudally: 7 cervical, 12 thoracic, 5
lumbar, 1 sacrum, coccyx. There has been identified a population variability in the final number of vertebra.

Correct Answer. d

Copyright © 2014 Delhi Academy of Medical Sciences, All Rights Reserved. 46/94
(97). Regarding the hip joint, which of the following statements is true?

a. Retinaculum attaches femur to hip

b. Inferior gluteal nerve supplies the abductors of the hip

c. Capsule is attached to the intertrochanteric line

d. Ilio psoas causes abduction after fracture neck femur

Solution. (c) Capsule is attached to the intertrochanteric line


Ref: Read the text below
Sol :
CAPSULE OF HIP JOINT
- This encloses the joint and is attached to the acetabulary labrum medially. Laterally, it is attached to the intertrochanteric line of the
femur in front and halfway along the posterior aspect of the neck of the bone behind.
- At its attachment to the intertrochanteric line in front, some of its fibers,
accompanied by blood vessels, are retlected upward along the neck as bands,
called retinacula. These blood vessels supply the head and neck of the
femur.

Correct Answer. c

(98). All are tributaries of thoracic duct except:

a. Left upper intercostals trunk

b. Right bronchomediastinal trunk

c. Left bronchomediastinal trunk

d. Right descending thoracic lymph trunk

Solution. (b) Right bronchomediastinal trunk


Ref.:Read the text below
Sol :
Tributaries of Thoracic Duct
- Bilateral descending thoracic lymph trunk
- Bilateral ascending lumbar lymph trunk
- Left upper intercostals trunk
- Mediastinal trunk
- Left subclavian trunk & left jugular trunk
- Left bronchomediastinal trunk

Correct Answer. b

Copyright © 2014 Delhi Academy of Medical Sciences, All Rights Reserved. 47/94
(99). Anconeusis supplied by:

a. Median nerve

b. Radial nerve

c. Musculocutaneus nerve

d. Posterior interosseous nerve

Solution. (b) Radial nerve


Ref.:Read the text below
Sol :
The following are branches of the radial nerve (including the superficial branch of the radial nerve and the deep branch of the radial
nerve/posterior interosseous nerve).
Cutaneous
Cutaneous innervation of the right upper extremity. Areas innervated by the radial nerve are colored in pink.
Cutaneous innervation by the radial nerve is provided by the following nerve branches:
- Posterior cutaneous nerve of arm (originates in axilla)
- Inferior lateral cutaneous nerve of arm (originates in arm)
- Posterior cutaneous nerve of forearm (originates in arm)
The superficial branch of the radial nerve provides sensory innervation to much of the back of the hand, including the web of skin
between the thumb and index finger.
Motor
Muscles of the posterior forearm. All the labelled muscles (that is, all the visible muscles except the ones on the dorsal hand and one at
top left) are innervated by the radial nerve, and represent all muscles innervated by the radial nerve except for the supinator.
Muscular branches of the radial nerve:
- Triceps brachii (lateral and medial heads)
- Anconeus
- Brachioradialis
- Extensor carpi radialis longus
Deep branch of the radial nerve:
- Extensor carpi radialis brevis
- Supinator
Posterior interosseous nerve (a continuation of the deep branch after the supinator):
- Extensor digitorum
- Extensor digiti minimi
- Extensor carpi ulnaris
- Abductor pollicis longus
- Extensor pollicis brevis
- Extensor pollicis longus
- Extensor indicis
The radial nerve (and its deep branch) provides motor innervation to the muscles in the posterior compartment of the arm and forearm,
which are mostly extensors.

Correct Answer. b

(100). Superior orbital fissure lies in which part of sphenoid bone?

a. In lesser wing

b. Between lesser and greater wing

c. In greater wing

d. Between greater wing and maxilla

Solution. (b) Between lesser and greater wing


Ref.: Read the text below
Sol :
Superior orbital fissure :
- Connects the orbit with cranial cavity
- Superior orbital fissure lies between the lesser and the greater wing of sphenoid Boundaries
- Medial : body of sphenoid
- Above : lesser wing of sphenoid
- Below : medial margin of greater wing of sphenoid
- Lateral : frontal bone

Correct Answer. b

Copyright © 2014 Delhi Academy of Medical Sciences, All Rights Reserved. 48/94
(101). Foot drop results as a result of injury to :

a. Deep peroneal nerve

b. Superficial peroneal nerve

c. Tibial nerve

d. Deltoid ligament

Solution. (a) Deep peroneal nerve


Ref: Read the text below
Sol :
- Foot drop is the dropping of the forefoot due to weakness, damage to the
peroneal nerve or paralysis of the muscles in the anterior portion of the lower leg. It is usually a symptom of a greater problem, not a
disease in itself.
- It is characterized by the inability or difficulty in moving the ankle and toes upward (dorsiflexion).
- The severity in foot drop can range from a temporary to permanent condition, depending on the extent of muscle weakness or paralysis.
It can occur unilaterally or bilaterally.

Correct Answer. a

(102). The inferior tibiofibular joint is a :

a. Synostosis

b. Synchondrosis

c. Syndesmosis

d. Saddle joint

Solution. (c) Syndesmosis


Ref:Read the text below
Sol:
FIBROUS JOINTS
Are joined by fibrous tissue.
Are of three types -
a) SYNDESMOSIS - the bones are connected by interosseous ligament. - eg. inferior tibiofibular joint, Intervertebral joints
b) SUTURES :Peculiar to skull & are immovable
c) GOMPHOSIS :is a peg & socket joint , eg. Tooth & socket joint

Correct Answer. c

(103). All cause retraction of scapula except?

a. Rhomboides major

b. Rhomboides minor

c. Trapezius

d. Levator scapula

Solution. (d) Levator scapula


Ref:Gray’s - 837
Sol:
- The levator scapulae and the rhomboids assist other scapular muscles in
controlling the position and movement of the scapula.
- Acting with trapezius, rhomboids retract the scapula, bracing back the
shoulder; with levator scapulae and pectoralis minor they rotate the scapula, depressing the point of the shoulder.
- With the cervical vertebral column fixed, levator scapulae acts with trapezius to elevate the scapula or to sustain a weight carried on
the shoulder; with the shoulder fixed, the muscle inclines the neck to the same side.

Correct Answer. d

Copyright © 2014 Delhi Academy of Medical Sciences, All Rights Reserved. 49/94
(104). A healthy young athlete sitting at table with knee at 90 degree flexion. He fully extends it. what will happen?

a. Movement of tibial tuberosity towards lateral border of patella

b. Movement of tibial tuberosity towards medial border of patella

c. Movement of tibial tuberosity towards centre of patella

d. No movement

Solution. (a) Movement of tibial tuberosity towards lateral border of


patella
Ref:Read the text below
Sol:
- When the foot if firmly planted on the ground when a person is standing, the femur is medially rotated on the tibia to lock & stabilize
the knee joint.
- However, if the foot is raised off the ground, the tibia may be laterally rotated on the femur to lock the knee joint

Correct Answer. a

(105). Posteriorglenohumeral instability is diagnosed by

a. Crank test

b. Fulcrum

c. Jerk test

d. Sulcus

Solution. (c) Jerk test


Ref: Turek's Orthopaedics: Principles and Their Application, 6th
EditionP.388
Rockwood fracture in adults 5thed
Sol:
GLENOHUMERAL INSTABILITY
- Glenohumeral instability is a common problem encountered by the
orthopaedic surgeon, particularly in young, active individuals..
- Jerk Test For the jerk test the patient sits with the arm internally rotated and flexed forward to 90 degrees. The examiner grasps the
elbow and axial loads the humerus in a proximal direction. While axial loading of the humerus is maintained, the arm is moved
horizontally across the body. In many patients with recurrent posterior instability this test will produce a sudden jerk when the humeral
head slides off the back of the glenoid. When the arm is returned to the original position of 90 degrees of abduction, a second jerk may
be observed—that of the humeral head returning to the glenoid.
- Push–Pull Test For the push–pull test the patient lies supine with the shoulder off the edge of the table. The arm is in 90 degrees of
abduction and 30 degrees of flexon.Standing next to the patient's hip, the examiner pulls up on the wrist with one hand while pushing
down on the proximal humerus with the other. The shoulders of normal, relaxed patients often will allow 50% posterior translation on
this test. Greater degrees of translation, apprehension, or reproduction of the patient's symptoms suggest pathologic posterior
instability. This test should be performed on both sides to compare the degrees of translation and the patient's response.

Correct Answer. c

Copyright © 2014 Delhi Academy of Medical Sciences, All Rights Reserved. 50/94
(106). The porto-systemic anastomoses involve the following veins of the portal group except:-

a. Inferior rectal veins

b. Left gastric vein

c. Superior rectal vein

d. Paraumbilical veins

Solution. (a) Inferior rectal veins


Ref:Read the text below
Sol:

Correct Answer. a

(107). Coeliac plexus is present

a. Anterior lateral and around the aorta

b. Posterior medial and around the aorta

c. Anterior medial to lumbar symp chain

d. Posterior lateral to lumbar symp chain

Solution. (a) Anterior lateral and around the aorta


Ref:GRAY’S ANATOMY 40TH 1045
Sol:
COELIAC PLEXUS
- The coeliac plexus is the largest major autonomic plexus, sited at the level of the twelfth thoracic and first lumbar vertebrae.
- It is a dense network uniting two large coeliac ganglia and surrounds the coeliac artery and the root of the superior mesenteric artery .
- It is posterior to the stomach and lesser sac, anterior to the crura of the
diaphragm and the commencement of the abdominal aorta, and lies between the suprarenal glands.
- The plexus and ganglia are joined by greater and lesser splanchnic nerves and branches from the vagus and phrenic nerves.
- The plexus extends as numerous secondary plexuses along adjacent arteries.
- The coeliac ganglia are irregular masses on each side of the coeliac trunk adjacent to the suprarenal glands.
- They lie anterior to the crura of the diaphragm.
- The right ganglion is posterior to the inferior vena cava, the left ganglion posterior to the origin of the splenic artery.
- The ipsilateral greater splanchnic nerve joins the upper part of each ganglion.
- The lower part of each ganglion forms a distinct subdivision usually termed the aorticorenal ganglion.
- This receives the ipsilateral lesser splanchnic nerve and gives origin to the majority of the renal plexus.
- It most commonly lies anterior to the origin of the renal artery.
- The coeliac plexus is connected to or gives rise to the phrenic, splenic,
hepatic, superior mesenteric, suprarenal, renal and gonadal plexuses.

Correct Answer. a

Copyright © 2014 Delhi Academy of Medical Sciences, All Rights Reserved. 51/94
(108). About Prostatic urethra all are true except:-

a. Trapezoid in cross section

b. Verumomentum is present in posterior wall

c. Opening of prostatic ducts lies in posterior wall

d. Relation of urethral crest in posterior wall

Solution. (a) Trapezoid in cross section


Ref: GRAY’S - 1295
Sol:
- The male urethra is18-20 cm long and extends from the internal orifice in the urinary bladder to the external opening, or meatus, at the
end of the penis. It may be considered in two parts .
- The relatively long anterior urethra (16 cm long) lies within the perineum (proximally) and the penis (distally) surrounded by the corpus
spongiosum and is functionally a conduit.
- The relatively short posterior urethra (4 cm) lies in the pelvis proximal to the corpus spongiosum and is acted upon by the urogenital
sphincter mechanisms and also acts as a conduit.
- The anterior urethra is subdivided into a proximal component, the bulbar urethra, which is surrounded by the bulbospongiosus and is
entirely within the perineum, and a pendulous or penile component, which continues on to the tip of the penis.
- The posterior urethra is divided into preprostatic, prostatic, and membranous segments.
- In the flaccid penis, the urethra as a whole presents a double curve .
- Except during the passage of fluid along it, the urethral canal is a mere slit: in transverse section, the slit is transversely arched in the
prostatic part, in the preprostatic and membranous portions it is stellate, in the bulbar and penile portions it is transverse, while at the
external orifice it is sagittal MEMBRANOUS PART
- The membranous part of the urethra is the shortest (c.1.5 cm), least
dilatable and, with the exception of the external orifice, the narrowest, section of the urethra.
PREPROSTATIC PART
- The preprostatic urethra is c.1-1.5 cm in length, extending almost vertically from the bladder neck to the superior aspect of the
verumontanum.
PROSTATIC PART
- The prostatic urethra is c.3-4 cm in length and tunnels through the substance of the prostate closer to the anterior than the posterior
surface of the gland.
- The urethra turns anteriorly as it passes through the prostate making an angle of 35.
- Throughout most of its length the posterior wall possesses a midline ridge, the urethral crest, which projects into the lumen causing it
to appear crescentic in transverse section.
- On each side of the crest there is a shallow depression, termed the prostatic sinus, the floor of which is perforated by the orifices of
c.15-20 prostatic ducts.
- An elevation, the verumontanum (colliculus seminalis), at about the middle of the length of the urethral crest, contains the slit-like
orifice of the prostatic utricle.
- On both sides of, or just within, this orifice are the two small openings of
the ejaculatory ducts.
- The prostatic utricle is a cul-de-sac c.6 mm long, which runs upwards and backwards in the substance of the prostate behind its median
lobe.
- Its walls are composed of fibrous tissue, muscular fibres and mucous
membrane; the latter is pitted by the openings of numerous small glands.

Correct Answer. a

Copyright © 2014 Delhi Academy of Medical Sciences, All Rights Reserved. 52/94
(109). Middle superior alveolar nerve is a branch of :

a. Palatine branch of maxillary

b. Nasal branch of maxillary

c. Infraorbital nerve

d. Mandibular nerve

Solution. (c) Infraorbital nerve


Ref: Gray’s - 699
Sol:
MAXILLARY NERVE
- The maxillary nerve is a sensory division of the trigeminal nerve.
- Most of the branches from the maxillary nerve arise in the pterygopalatine fossa.
- It gives rise to the zygomatic and infraorbital nerves that pass into the orbit through the inferior orbital fissure and two others that pass
through the pterygopalatine ganglion without synapsing and are distributed to the nose, palate and pharynx. Zygomatic nerve
- The zygomatic nerve is located close to the base of the lateral wall of the orbit. It soon divides into two branches, the
zygomaticotemporal and the
zygomaticofacial nerves, which run for only a short distance in the orbit
before passing onto the face through the lateral wall of the orbit.
- They may either enter separate canals within the zygomatic bone or the zygomatic nerve itself may enter the bone before dividing.
- The zygomaticotemporal nerve exits the zygomatic bone on its medial surface, and pierces the temporal fascia to supply the skin over
the temple.
- It also gives a branch to the lacrimal nerve which may carry parasympathetic fibres to the lacrimal gland .
Infraorbital nerve
- The infraorbital nerve initially lies in the infraorbital groove on the floor of the orbit.
- The infraorbital nerve supplies the skin of the lower eyelid, possibly the
conjunctiva, and skin over the upper jaw, and also provides the middle and anterior superior alveolar nerves.

Correct Answer. c

Copyright © 2014 Delhi Academy of Medical Sciences, All Rights Reserved. 53/94
(110). A 12-year-old boy has noted facial pain with fever for the past 3 days. On physical examination there is swelling and tenderness of the structure
identified in Image E. Infection with which of the following organisms is most likely to produce these findings?

a. Aspergillus fumigate

b. Plasmodium falciparum

c. Mumps virus

d. Candida albicans

Solution. (c) Plasmodium falciparum


Ref:Read the text below
Sol :
The parotid glands can become inflamed (parotitis) with mumps infection,
which is a common childhood infection.

Correct Answer. c

Copyright © 2014 Delhi Academy of Medical Sciences, All Rights Reserved. 54/94
(111). All are External laryngeal membrane except

a. Cricothyroid

b. Thyrohyoid

c. Cricotracheal

d. Hyoepiglotic

Solution. (a) Cricothyroid


Ref: Gray’s - 636
Sol:
EXTRINSIC LIGAMENTS AND MEMBRANES
Thyrohyoid membrane
- The thyrohyoid membrane is a broad, fibroelastic layer, attached below to the superior border of the thyroid cartilage lamina and the
front of its superior cornua, and above to the superior margin of the body and greater cornua of the hyoid. Hyo- and thyroepiglottic
ligaments
- The epiglottis is attached to the hyoid bone and thyroid cartilage by the extrinsic hyoepiglottic and intrinsic thyroepiglottic ligaments
respectively.
Cricotracheal ligament
- The cricotracheal ligament unites the lower cricoid border to the first tracheal cartilage, and is thus continuous with the perichondrium
of the trachea
INTRINSIC LIGAMENTS AND MEMBRANES
- The fibroelastic membrane of the larynx lies within the cartilaginous skeleton of the larynx, beneath the laryngeal mucosa.
- It is interrupted on both sides of the larynx by a horizontal cleft between the vestibular and vocal ligaments.
- Its upper part, the quadrangular membrane, extends between the arytenoid cartilages and the sides of the epiglottis.
- Its lower part forms the cricovocal membrane, which connects the thyroid, cricoid and arytenoid cartilages.
Quadrangular membrane
- Each quadrangular membrane passes from the lateral margin of the
epiglottis to the arytenoid cartilage on its own side.
- It is often poorly defined. The upper and lower borders of the membrane are free.
- The upper border slopes posteriorly to form the aryepiglottic ligament,
which constitutes the central component of the aryepiglottic fold.
- It is less defined in its upper portion.
- Posteriorly it passes through the fascial plane of the oesophageal
suspensory ligament, and helps to form a median corniculopharyngeal ligament which extends into the submucosa adjacent to the cricoid
cartilage.
- This ligament may exert vertical traction. The lower border forms the vestibular fold. The cuneiform cartilages lie within the
aryepiglottic folds.
Cricothyroid ligament and cricovocal membrane
- The cricothyroid ligament is composed mainly of elastic tissue. It consists of two parts: an anterior part, the anterior (median)
cricothyroid ligament, and a lateral part, the cricovocal membrane.
Cricothyroid membrane and anterior (median) cricothyroid ligament
- The cricothyroid membrane passes upwards from the upper border of the cricoid cartilage to the lower border of the thyroid cartilage.
- Anteriorly, it is thickened to form the anterior (median) cricothyroid
ligament, which is broader below and narrower above.

Correct Answer. a

Copyright © 2014 Delhi Academy of Medical Sciences, All Rights Reserved. 55/94
(112). Morgagni hernia presents most commonly on

a. Left posterior

b. Right anterior

c. Right posterior

d. Left anterior

Solution. (b) Right anterior


Ref:Read the text below
Sol:
- Congenital diaphragmatic hernia (CDH) represents one of the most enigmatic diseases encountered in pediatric surgery.
- Most CDH defects are on the left side (80%); however, up to 20% may occur on the right side.
- A CDH may also be bilateral, but this is distinctly rare.
Pathogenesis
- The cause for CDH is unknown, but it is thought to result from failure of
normal closure of the pleuroperitoneal canal in the developing embryo.
- As a result, abdominal contents herniate through the resultant defect in the posterolateral diaphragm and compress the ipsilateral
developing lung.
- The posterolateral location of this hernia is known as Bochdalek's hernia
and is distinguished from the congenital hernia of the anteromedial, retrosternal diaphragm, which is known as Morgagni's hernia.
Clinical Presentation
- The most frequent clinical presentation of CDH is respiratory distress due to severe hypoxemia.
- The infant appears dyspneic, tachypneic, and cyanotic, with severe retractions. The anteroposterior diameter of the chest may be large,
and the abdomen may be scaphoid.

Correct Answer. b

(113). About sternocleidomastoid tumor all are true except –

a. Always associated with breech

b. Spontaenous resolution in most cases

c. Two-third have palpable neck mass at birth

d. Uncorrected cases develops plagiocephaly

Solution. (d) Uncorrected cases develops plagiocephaly


Ref:Read the text below
Sol:
- Torticollis presents with a hard, nontender, fibrotic mass within the
sternocleidomastoid muscle.
- It may be present at birth but is usually not noticed until the second to
sixth weeks of life.
- The mass appears with equal frequency in both sexes and on each side of the neck.
- Rarely, there is more than one mass in the muscle or both sternocleidomastoid muscles are involved.
- A history of breech delivery is present in 20–30% of these children.
Clinical Findings
- Torticollis is manifested when the sternocleidomastoid muscle is shortened and the mastoid process on the involved side is pulled down
toward the clavicle and manubrium.
- As a result, the head is abducted to the ipsilateral side and rotated to the contralateral side (toward the opposite shoulder).
- The shoulder on the affected side is raised, and there may be cervical and thoracic scoliosis.
- Passive rotation of the head to the side of the involved muscle will be
resisted and limited to varying degrees, and the muscle will appear as a protuberant band
- Because of persistent pressure when the patient is recumbent, the ipsilateral face and contralateral occiput will be flattened.
- Facial hemihypoplasia and plagiocephaly (flattening of the ipsilateral posterior skull) occurs in untreated cases, usually within 6
months.
Treatment
- Surgery is rarely necessary for this disorder.
- Torticollis is treated with active range of motion exercises.
- The child's shoulders are held flat to a table and the head is tilted and rotated in a full range of motion.
- This procedure should be performed at least four times a day, usually for
2–3 months.
- The firm "tumor" often disappears well before the torticollis is cured. If the muscle continues to become progressively shortened, with
facial and occipital skull deformity, both heads of the sternocleidomastoid muscle should be divided through a small transverse incision
just above the clavicle.
- This procedure does not reverse the bony changes that have already developed but prevents progression of the process.

Correct Answer. d

Copyright © 2014 Delhi Academy of Medical Sciences, All Rights Reserved. 56/94
(114). Which of the following statement about Paneth cellsis true:

a. Rich in rough endoplasmic reticulum

b. High zinc content

c. Foamy cytoplasm

d. None.

Solution. (b) High zinc content


Ref: Gray’s -1139
Sol:
Paneth cells
- Paneth cells are numerous in the deeper parts of the intestinal crypts,
particularly in the duodenum.
- They are rich in zinc and contain large acidophilic granules that stain strongly with eosin or phosphotungstic haematoxylin.
- Paneth cells secrete lysozyme, a highly specific antibacterial enzyme, and other defensive proteins (defensins) such as tumour necrosis
factor alpha (TNF-α), which protect the intestinal luminal surface.

Correct Answer. b

(115). The function of 8th cranial nerveis related to

a. Smell

b. Taste

c. Touch

d. Balance

Solution. (d) Balance


Ref:Read the text below
Sol:
- The vestibulocochlear nerve (also known as the auditory or acoustic nerve) is the eighth of twelve cranial nerves, and is responsible for
transmitting sound and equilibrium (balance) information from the inner ear to the brain

Correct Answer. d

(116). Following structures are present above the perineal membrane in females except :

a. Sphincter urethrae

b. Compressor urethrae

c. Sphincter urethrovaginalis

d. Bartholin's gland

Solution. (d) Bartholin's gland


Ref:Read the text below
Sol:
- The glands in females which are homologous to bulbourethral glands in males, are the greater vestibular glands or Bartholin's glands.
- They are round or oval, reddish-yellow bodies flanking the vaginal orifice.
- They are situated inferior to the perineal membrane in females.

Correct Answer. d

Copyright © 2014 Delhi Academy of Medical Sciences, All Rights Reserved. 57/94
(117). The trapezius attaches to which of the following regions of the clavicle?

a. Lateral one-third of the clavicle

b. Conoid tubercle

c. Subclavian groove

d. Trapezoid line

Solution. (a) Lateral one-third of the clavicle


Ref: Read the text below
Sol :
- The trapezius attaches to the lateral third of the clavicle, acromion, and
spine of the scapula.
- The deltoid attaches to the deltoid tubercle, the conoid ligament attaches to the conoid tubercle, the subclavius attaches to the
subclavian groove, and the trapezoid ligament attaches to the trapezoid line.

Correct Answer. a

(118). Which of the following muscles attaches to the coracoid process of the scapula?

a. Pectoralis minor

b. Triceps brachii

c. Brachialis

d. Pectoralis major

Solution. (a) Pectoralis minor


Ref: Read the text below
Sol :
- The pectoralis minor, biceps brachii (short head), and coracobrachialis attach to the coracoids process of the scapula.

Correct Answer. a

(119). All of the following are medial rotators of the arm except

a. Latissimus dorsi

b. Teres major

c. Subscapularis

d. Infraspinatus

Solution. (d) Infraspinatus


Ref: Read the text below
Sol :
- The latissimus dorsi, teres major, and subscapularis medially rotate the arm.
- The infraspinatus and teres minor rotate the arm laterally.
- The deltoid is unique in that its anterior part rotates the arm medially, and its posterior part rotates the arm laterally.

Correct Answer. d

Copyright © 2014 Delhi Academy of Medical Sciences, All Rights Reserved. 58/94
(120). Multiple spontaneous idiopathic and symmetrical fractures are characteristic of :

a. Stress fracture

b. Milkman’s syndrome

c. Rickets

d. Osteoporosis

Solution. (2) Milkman’s syndrome


Ref: Read the text below
Sol :
- Decreased tubular reabsorption of phosphate, resulting in osteomalacia which gives a peculiar striped appearance (multiple
pseudofractures) to the bones in roentgenograms.
- Also known as Looser-Milkman syndrome.

Correct Answer. b

(121). Histology of myositis ossificans mimics :

a. Hemophilic arthropathy

b. Osteosarcoma

c. Osteoclastoma

d. Fibrosarcoma

Solution. (2) Osteosarcoma


Ref: Read the text below
Sol :
- Myositis ossificans differs histological from extraosseous osteosarcoma by zone phenomenon characterized by well oriented bone
speckles separated by benign appearing stroma near outer margin of lesion

Correct Answer. b

(122). A patient is asked to place the hands posteriorly on the hips and to push the elbows posteriorly against resistance. Which muscle is being
tested?

a. Levator scapulae

b. Rhomboid

c. Trapezius

d. Latissimus dorsi

Solution. (b) Rhomboid


Ref: Read the text below
Sol :
- To test the rhomboids, the patient is asked to place the hands posteriorly on the hips and to push the elbows posteriorly against
resistance

Correct Answer. b

Copyright © 2014 Delhi Academy of Medical Sciences, All Rights Reserved. 59/94
(123). Which rotator cuff muscle does not rotate the humerus?

a. Supraspinatus

b. Infraspinatus

c. Teres minor

d. Subscapularis

Solution. (a) Supraspinatus


Ref: Read the text below
Sol :
- The supraspinatus is the only rotator cuff muscle that does not rotate the
humerus.

Correct Answer. a

(124). ANTERIOR ETHMOIDAL NERVE SUPPLIES ALL EXCEPT :

a. DURA MATER IN ANTERIOR CRANIAL FOSSA

b. NASAL CAVITY INTERNAL

c. ETHMOIDAL CELLS

d. MAXILLARY SINUS LINING

Solution. (a) DURA MATER IN ANTERIOR CRANIAL FOSSA


Ref: Gray’s 40th ed 699
Sol:
- The nasociliary nerve is intermediate in size between the frontal and
lacrimal nerves, and is more deeply placed in the orbit, which it enters through the common tendinous ring, lying between the two rami
of the oculomotor nerve.
- It crosses the optic nerve with the ophthalmic artery and runs obliquely below superior rectus and superior oblique to reach the medial
orbital wall.
- Here, as the anterior ethmoidal nerve, it passes through the anterior ethmoidal foramen and canal and enters the cranial cavity.
- It runs forwards in a groove on the upper surface of the cribriform plate beneath the dura mater and descends through a slit lateral to
the crista galli into the nasal cavity, where it occupies a groove on the internal surface of the nasal bone and gives off two internal nasal
branches
- The medial internal nasal nerve supplies the anterior septal mucosa, and the lateral internal nasal nerve supplies the anterior part of
the lateral nasal wall.
- The anterior ethmoidal nerve emerges, as the external nasal nerve , at the lower border of the nasal bone, and descends under the
transverse part of nasalis to supply the skin of the nasal ala, apex and vestibule.
- The nasociliary nerve has connections with the ciliary ganglion and has long ciliary, infratrochlear and posterior ethmoidal branches.
- The ramus communicans to the ciliary ganglion usually branches from the nerve as it enters the orbit lateral to the optic nerve.
- It is sometimes joined by a filament from the internal carotid sympathetic plexus or from the superior ramus of the oculomotor nerve as
it enters the posterosuperior angle of the ganglion.
- Two or three long ciliary nerves branch from the nasociliary nerve as it crosses the optic nerve .
- They accompany the short ciliary nerves and pierce the sclera near the
attachment of the optic nerve.

Correct Answer. a

Copyright © 2014 Delhi Academy of Medical Sciences, All Rights Reserved. 60/94
(125). Appendices epiploicae are present in all of the following except:

a. Caecum

b. Appendix

c. Sigmoid colon

d. Rectum

Solution. (d) Rectum


Ref:Gray’s 40 1178
Sol:
- The haustrations of the colon are often absent in the caecum proximal to
the origin of the ascending colon and are often relatively sparse in the ascending and proximal transverse colon.
- In these regions the taeniae coli are usually thin and occupy only a small percentage of the circumference of the colon.
- There are few if any appendices epiploicae on the serosal surface of the
caecum, and only a limited number on the surface of the ascending colon.
- The haustrations become more pronounced from the middle of the transverse colon to the distal portion of the descending colon: the
sigmoid colon is often characterized by marked sacculation.
- The width of the taeniae coli remains fairly constant throughout the length of colon but the number of appendices usually increases,
becoming most numerous in the sigmoid colon where they can be fairly large in the obese individual.
- The taeniae are located in fairly constant positions beneath the serosal
surface of the colon except in the transverse colon.
- They are oriented anteriorly, opposite the midline of the mesenteric attachment on the anti-mesenteric aspect of the colon (taenia
libera), posterolaterally (taenia omentalis) and posteromedially (taenia mesocolica) midway between the taenia libera and the mesentery.
- In the caecum and descending colon, which are partly retroperitoneal structures, the posterolateral taenia is often obscured from view
by the peritoneal reflection onto the colonic wall.
- In the transverse colon, the taeniae are rotated through 90° - anterior being inferior, posteromedial being posterior and posterolateral
being superior - as a consequence of the mobility and dependent position of this part of the colon.
- The taeniae coli broaden to occupy more of the circumference of the
sigmoid colon in its distal portion and by the level of the rectosigmoid junction have widened to form distinct anterior and posterior
bands, which unite to form a complete longitudinal muscle covering for the rectum.
- The rectum therefore has no external sacculation and no serosal
appendices epiploicae.

Correct Answer. d

(126). Which is not supplied by pelvic splanchnic nerve?

a. Appendix

b. Rectum

c. Uterus

d. Urinary Bladder

Solution. (a) Appendix


Ref: Gray’s 40th ed 238
Sol:
- Pelvic splanchnic nerves to the pelvic viscera travel in anterior rami of the second, third and fourth sacral spinal nerves.
- These nerves unite with branches of the sympathetic pelvic plexuses. Minute ganglia occur at the points of union and in the visceral
walls, and sacral preganglionic parasympathetic fibres relay synaptically in these ganglia.
- The pelvic splanchnic nerves are motor to the muscle of the rectum and bladder wall but inhibitory to the vesical sphincter.
- They supply vasodilator fibres to the erectile tissue of the penis and clitoris and are probably also vasodilator to the testes, ovaries,
uterine tubes and uterus.
- Filaments from the pelvic splanchnic nerves ascend in the hypogastric plexus and are visceromotor to the sigmoid and descending
colon, the left colic flexure and terminal transverse colon.

Correct Answer. a

Copyright © 2014 Delhi Academy of Medical Sciences, All Rights Reserved. 61/94
(127). A 10-year-old boy has obstruction of the structure identified in Image F. Which of the following is most likely to cause this obstruction

a. Blood clot

b. Hyperostosis

c. Infection

d. Wax

Solution. (d) Wax


Ref:Read the text below
Sol :
- The ceruminous glands of the external auditory canal, in some persons, produce excessive amounts of ear wax that can occlude this
canal and diminish hearing.
- Gentle irrigation with warm water is a good means to remove the wax.

Correct Answer. d

(128). The arachnoid villi responsible for cerebrospinal fluid absorption protrude mainly in the:

a. Superior saggital sinus

b. Inferior saggital sinus

c. Straight sinus

d. Transverse sinus

Solution. (a) Superior saggital sinus


Ref: Read the text below
Sol:
- The arachnoid villi are projections of the arachnoid matter into the venous sinuses. These villi are most numerous in the superior
saggital sinus.
- CSF is produced in the brain by modified ependymal cells in the choroid plexus (approx. 50-70%), and the remainder is formed around
blood vessels and along ventricular walls.
- It circulates from the lateral ventricles to the foramen of Monro, third ventricle, aqueduct of Sylvius, fourth ventricle, foramina of
Magendie and Luschka; subarachnoid space over brain and spinal cord; reabsorption into venous sinus blood via arachnoid granulations.

Correct Answer. a

Copyright © 2014 Delhi Academy of Medical Sciences, All Rights Reserved. 62/94
(129). All of the following are derivatives of the neural crest, except:

a. Melanocyte

b. Adrenal Medulla

c. Sympathetic Ganglia

d. Cauda Equina

Solution. (d) Cauda Equina


Ref: Read the text below
Sol:
Cauda Equina
The following are structures derived from the neural crest:
1. Neurons:
- Spinal posterior nerve root ganglia (dorsal root ganglia)
- Sensory ganglia of the 5th, 7th, 8th, 9th and 10th cranial nerves
- Sympathetic ganglia
2. Pia mater and Arachnoid mater: Not Dura mater
3. Schwann cells
4. Adrenal Medulla, Chromaffin tissue
5. Melanoblasts, mesenchyme of dental papilla

Correct Answer. d

(130). All of the following features are seen in neurons from dorsal root ganglia,except:

a. They have centrally located nuclei

b. They are derived from neural crest cells

c. They are multipolar

d. They contain lipofuscin granules

Solution. (c) They are multipolar


Ref: Read the text below
Sol:
- They have centrally located nuclei as in the picture
- The dorsal root ganglia develops in the embryo from neural crest cells.
- These neurons are of the pseudo-unipolar type, meaning they have an axon with two branches that act as a single axon, often referred
to as a distal process and a proximal process.

Correct Answer. c

(131). Occlusion of the anterior descending branch of left coronary artery will lead to infarction of which area?

a. Anterior wall of left ventricle

b. Posterior part of the interventricular septum

c. Lateral part of the heart

d. Inferior surface of the right ventricle

Solution. (a) Anterior wall of left ventricle


Ref: Read the text below
Sol:
The following is a description of the frequencies of critical narrowing and
thrombosis of each of the 3 main arterial trunks and the corresponding
sites of myocardial lesions:
- Left anterior descending coronary artery (40-50%, most common) - Lesions in anterior wall of left ventricle near apex and anterior 2/3
of interventricular septum.
- Right coronary artery (30-40%) - Lesions in inferior or posterior wall of left ventricle, posterior 1/3 of interventricular septum, and
posterior right ventricular free wall in some cases.
- Left circumflex coronary artery (15-20%) - Lesions in lateral wall of left ventricle

Correct Answer. a

Copyright © 2014 Delhi Academy of Medical Sciences, All Rights Reserved. 63/94
(132). On examination following surgical removal of a firm nodular cancer swelling in the right breast and exploration of the right axilla, a
patient is
found to have a winged scapula. This occurred due to injury to the:

a. Subscapular muscle

b. Coracoid process of scapula

c. Long thoracic nerve

d. Circumflex scapular artery

Solution. (c) Long thoracic nerve


Ref: Read the text below
Sol:
- In the normal individual, the pull of the serratus anterior muscle keeps the medial border against the thoracic wall. However, when
there is injury to the long thoracic nerve which supplies the serratus anterior muscle, there is winging of scapula, which is excessive
prominence of the medial border of the scapula.
- Actions of the serratus anterior muscle are:
1. It pulls the scapula forwards around the chest wall to protect the upper limb, as in pushing and punching actions
2. It rotates the scapula so that the glenoid cavity is turned upwards, which allows abduction of the arm above 90 degree.
In paralysis of the serratus anterior muscle, when movements like pushing, punching and raising arm above 90 degree is attempted,
winging of scapula occurs.
- Long Thoracic nerve is also known as “Nerve of Bell”.
- Serratus anterior muscle arises by eight digitations from the upper eight ribs.

Correct Answer. c

(133). A patient with external haemorrhoids develops pain while passing stools. Nerve mediating this pain is:

a. Hypogastric nerve

b. Pudendal nerve

c. Splanchnic visceral nerve

d. Sympathetic plexus

Solution. (b) Pudendal nerve


Ref: Read the text below
Sol:
- External haemorrhoids occur below the pectinate line and are very painful. This region is supplied by Inferior Rectal nerve (S2, S3, S4),
a somatic nerve, which in turn is a branch of the pudendal nerve.
- In contrast, the anal canal above the pectinate line is supplied by
Sympathetic (Inferior Hypogastric plexus, L1 and L2) and Parasympathetic (Pelvic Splanchnic, S2, S3, S4)
-They carry pain fibres from the internal haemorrhoids.

Correct Answer. b

(134). Fundus of the stomach is supplied by

a. Left gastric artery

b. Right gastric artery

c. Short gastric artery

d. Right gastro-epiploic artery

Solution. (c) Short gastric artery


Ref: Read the text below
Sol:
- The lesser curvature of the stomach is supplied by the right gastric
artery inferiorly, and theleft gastric artery superiorly, which also supplies the cardiac region.
- The greater curvature is supplied by the right gastroepiploic artery inferiorly and the (c) Short gastric artery

Correct Answer. c

Copyright © 2014 Delhi Academy of Medical Sciences, All Rights Reserved. 64/94
(135). The deep artery of the arm accompanies which of the following before passing around the body of the humerus?

a. Radial nerve

b. Musculocutaneous nerve

c. Median nerve

d. Ulnar nerve

Solution. (a) Radial nerve


Ref: Read the text below
Sol :
- The deep artery of the arm accompanies the radial nerve through the radial groove and passes around the body of the humerus

Correct Answer. a

(136). Which of the following intrinsic muscles of the thumb is most likely injured in Bennett's fracture?

a. Abductor pollicis brevis

b. Flexor pollicis brevis (superficial head)

c. Opponens pollicis

d. Adductor pollicis

Solution. (c) Opponens pollicis


Ref:Read the text below
Sol:
- The opponens pollicis inserts on the first metacarpal.
- All other intrinsic muscles of the thumb, including the abductor pollicis brevis, the flexor pollicis brevis, and the adductor pollicis
muscles, insert on the proximal phalanges.

Correct Answer. c

Copyright © 2014 Delhi Academy of Medical Sciences, All Rights Reserved. 65/94
(137). Structure not developing in mesentry of stomach

a. Liver

b. Kidney

c. Pancreas

d. Spleen

Solution. (b) Kidney


Ref:Langman's Medical Embryology 10thed pg 222,212,215
Sol:
About Mesentry
- Portions of the gut tube and its derivatives are suspended from the dorsal and ventral body wall by mesenteries, double layers of
peritoneum that enclose an organ and connect it to the body wall.
- Such organs are called intraperitoneal, whereas organs that lie against the posterior body wall and are covered by peritoneum on their
anterior
surface only (e.g., the kidneys) are considered retroperitoneal.
- Peritoneal ligaments are double layers of peritoneum (mesenteries) that pass from one organ to another or from an organ to the body
wall. Mesenteries and ligaments provide pathways for vessels, nerves, and lymphatics to and from abdominal viscera
- Initially the foregut, midgut, and hindgut are in broad contact with the mesenchyme of the posterior abdominal wall.
- By the fifth week, however, the connecting tissue bridge has narrowed, and the caudal part of the foregut, the midgut, and a major part
of the hindgut are suspended from the abdominal wall by the dorsal mesentery, which extends from the lower end of the esophagus to
the cloacal region of the hindgut.
- In the region of the stomach, it forms the dorsal mesogastrium or greater omentum; in the region of the duodenum, it forms the dorsal
mesoduodenum; and in the region of the colon, it forms the dorsal mesocolon. Dorsal mesentery of the jejunal and ileal loops forms the
mesentery proper.
- Ventral mesentery, which exists only in the region of the terminal part of the esophagus, the stomach, and the upper part of the
duodenum, is derived from the septum transversum. About Pancreas
- The pancreas is formed by two buds, dorsal and ventral, originating from the endodermal lining of the duodenum.
- Whereas the dorsal pancreatic bud is in the dorsal mesentery, the ventral pancreatic bud is close to the bile duct About Liver
- Growth of the liver into the mesenchyme of the septum transversum divides the ventral mesentery into (a) the lesser omentum,
extending from the lower portion of the esophagus, the stomach, and the upper portion of the duodenum to the liver and (b) the falciform
ligament, extending from the liver to the ventral body wall in the fifth week of development, the spleen primordium appears as a
mesodermal proliferation between the two leaves of the dorsal mesogastrium.
About Spleen
- With continued rotation of the stomach, the dorsal mesogastrium lengthens, and the portion between the spleen and dorsal midline
swings to the left and fuses with the peritoneum of the posterior abdominal wall.

Correct Answer. b

(138). The superficial muscle fibers of the external abdominal oblique arising from the middle to lower ribs interdigitate with which of the
following
muscles?

a. Internal abdominal oblique

b. Serratus anterior

c. Rectus abdominis

d. Pyramidalis

Solution. (b) Serratus anterior


Ref:Essential Clinical Anatomy by Moore p 180.
Sol:
- The superficial fibers of the external abdominal oblique arising from the
middle and lower ribs interdigitate with only those of the serratus anterior

Correct Answer. b

Copyright © 2014 Delhi Academy of Medical Sciences, All Rights Reserved. 66/94
(139). Which of the following structures is located in the posterior mediastinum?

a. Lungs

b. Heart

c. Azygos vein

d. Superior vena cava

Solution. (c) Azygos vein


Ref:Essential Clinical Anatomy by Moore-2
Sol:
- The posterior mediastinum contains the thoracic aorta, thoracic duct, azygos and hemiazygos veins, esophagus, and thoracic
sympathetic trunks.

Correct Answer. c

(140). Which of the following does not supply the breast with blood?

a. Lateral thoracic artery

b. Thoracoacromial artery

c. Internal thoracic artery

d. Costoclavicular artery

Solution. (d) Costoclavicular artery


Ref:Essential Clinical Anatomy by Moore-75
Sol:
- The breast is supplied by the internal thoracic, lateral thoracic, thoracoacromial, and posterior intercostal arteries

Correct Answer. d

(141). Which of the following are innervated by dorsal rami?

a. Levatores costarum

b. External intercostals

c. Internal intercostals

d. Subcostals

Solution. (a) Levatores costarum


Ref:Essential Clinical Anatomy by Moore-84
Sol:
- The Levatores costarum, twelve in number on either side, are small tendinous and fleshy bundles, which arise from the ends of the
transverse processes of the seventh cervical and upper eleven thoracic vertebrae
- They pass obliquely downward and laterally, like the fibers of the Intercostales externi, and each is inserted into the outer surface of the
rib immediately below the vertebra from which it takes origin, between the tubercle and the angle (Levatores costarum breves).
- Each of the four lower muscles divides into two fasciculi, one of which is inserted as above described; the other passes down to the
second rib below its origin (Levatores costarum longi).
- The levatores costarum are innervated by the dorsal primary rami of C8–T11.

Correct Answer. a

Copyright © 2014 Delhi Academy of Medical Sciences, All Rights Reserved. 67/94
(142). The anterolateral abdominal wall is bounded by all of the following structures except

a. Cartilages of the 7th through 10th ribs

b. Linea alba

c. Xiphoid process

d. Inguinal ligament

Solution. (b) Linea alba


Ref:Essential Clinical Anatomy by Moore p 178.
Sol:
- The anterolateral abdominal wall includes the cartilages of the 7th–10th ribs and xiphoid process superiorly.
- Inferiorly it is bounded by the inguinal ligament and the pelvic bone

Correct Answer. b

(143). Myositis ossificanscontain :

a. Bone

b. Cartilage

c. Capillaries

d. All

Solution. (b) Cartilage


Ref: Read the text below
Sol :
Myositis ossificans (MO) is a benign process characterised by heterotopic
ossification usually with large muscles. Myositis ossificans is essentially extraosseous bone formation (without inflammation) which
occurs in muscle. It has a zonal organisation :
- peripheral well organized mature lamellar bone
- intermediate osteoid region
- central immature non-ossified cellular focus
Unfortunately the histologically of myositis ossificans can appear reminiscent of osteosarcoma, and thus can actually lead to
inappropriate management.

Correct Answer. b

(144). Gastric sympathetic efferent nerves emerge from spinal cord segments:

a. T3 though T5

b. T2 through T4

c. T5 through T9

d. T7 though T12

Solution. (c) T5 through T9


Ref:Read the text below
Sol :
- Gastric sympathetic efferents emerge at the level of T5 through T9.
- They are preganglionic fibers that eventually synapse at the celiac ganglion.

Correct Answer. c

Copyright © 2014 Delhi Academy of Medical Sciences, All Rights Reserved. 68/94
(145). The arterial blood supply to the stomach is derived from all of the following except :

a. Right hepatic artery

b. Common hepatic artery

c. Superior mesenteric artery

d. Left gastric artery

Solution. (a) Right hepatic artery


Ref:Read the text below
Sol :
- Most of the stomach is supplied by branches of the celiac artery (i.e. splenic, left gastric, and common hepatic arteries).
- The right, middle, and left hepatic arteries supply the liver and biliary tree.

Correct Answer. a

(146). Parotid gland is divided into a superficial and a deep lobe by the facial nerve and its branches. From superficial to deep, the structures
traversing the gland are : facial nerve and its branches, retromandibular vein, and external carotid artery. Which of the following is related to the
anteromedial surface of parotid gland?

a. Auriculotemporal nerve

b. External carotid artery

c. Posterior belly of digastrics muscle

d. Medial pterygoid

Solution. (d) Medial pterygoid


Ref.:Read the text below
Sol :
Parotid Gland :
- It is the largest salivary gland.
- The gland has 4 surfaces: superior, superficial, anteromedial & posteromedial surface
- It has 3 borders : anterior, posterior, medial

-Relations

Correct Answer. d

Copyright © 2014 Delhi Academy of Medical Sciences, All Rights Reserved. 69/94
(147). A patient complains of persistent numbness of the chin, lower lip and lower teeth. She further indicates that she has difficulty in
chewing.
Radiographic studies of the head demonstrate a small discrete mass in the infratemporal fossa. Which nerve has been compromised by
the mass ?

a. Buccal nerve

b. Lingual nerve

c. Auriculotemporal nerve

d. Inferior alveolar nerve

Solution. (d) Inferior alveolar nerve


Ref: Read the text below
Sol :
- Inferior alveolar nerve has been compromised by the mass.
- Inferior alveolar nerve enter mandibular foramen on the medial aspect of body of mandible and emerge through mental foramen and
supply areas of chin & lower lip.

Correct Answer. d

(148). Which of the following is an example of saddle joint ?

a. Carpo-metacarpal joint of the thumb

b. Elbow

c. Ankle Joint

d. Knee joint.

Solution. (a) Carpo-metacarpal joint of the thumb


Ref: Read the text below
Sol :
SADDLE JOINT
A saddle joint allows movement back and forth and up and down, bot does not allow for rotation like a ball and socket joint.

Correct Answer. a

Copyright © 2014 Delhi Academy of Medical Sciences, All Rights Reserved. 70/94
(149). Eye ball movements are controlled by all of the following cranial nerves except :

a. Optic nerve

b. Abducens nerve

c. Trochlear nerve

d. Oculomotor nerve

Solution. (a) Optic nerve


Ref: Read the text below
Sol :
- Eye movements are controlled by muscles innervated by cranial nerves III, IV and VI. In this chapter, the testing of these cranial nerves
will be discussed.
- The most common symptom of damage to these nerves is double vision. The oculomotor nerve has the additional function of control of
the pupil and therefore this will be discussed here as well.
- Eye movements are carefully controlled by other systems. Cranial nerves III, IV, VI. Ocular Motility Oculomotor function can be divided
into two categories: (1) extraocular muscle function and (2) intrinsic ocular muscles (controlling the lens and pupil). The extraocular
muscles include: the medial, inferior, and superior recti, the inferior oblique, and levator palpebrae muscles, all innervated by the
oculomotor nerve (III);
the superior oblique muscle, innervated by the trochlear nerve (IV); and the lateral rectus muscle, innervated by the abducens nerve (VI).
The intrinsic eye muscles are innervated by the autonomic systems and include the iris sphincter and the ciliary muscle (innervated by
the parasympathetic component of cranial nerve III), and the radial pupillodilator muscles (innervated by the ascending cervical
sympathetic
system with its long course from spinal segments T1 through T3).

Correct Answer. a

(150). Broad thumb is seen in all except :

a. Acrocephalopolysyndactyly

b. Oto palato digital syndrome

c. Rubinson taybi syndrome

d. Fanconi anemia.

Solution. (d) Fanconi anemia.


Ref: Read the text below
Sol :
- Fanconi syndrome is associated with small or short thumb with radial ray abnormality along with pancytopenia.
- Causes of short or small thumbs are : holt oram syndrome, brachydactyly C or D, fetal hydantoin, fibrodysplasia ossificans progressiva,
cornelia de laange syndrome and Fanconi syndrome.
- Causes of absent thumbs are : fanconi syndrome, Poland syndrome,
thalidomide, trisomy 18
- Causes of large thumbs are : Klippel trenaunay weber syndrome, macrodystrophia lipomatosa, maffucci’s syndrome, neurofibromatosis.

Correct Answer. d

(151). Short metacarpal or metatarsal is seen in

a. Turner’s syndrome

b. Down’s syndrome

c. Marfan’s syndrome

d. Ellis van Creveld syndrome

Solution. (a) Turner’s syndrome


Ref: Poznanski -209-62
Sol :
- Short metacarpal or metatarsal are seen in idiopathic conditions, post traumatic, sickle cell anemia, tuner’s syndrome,
Pseudohypoparathyroidism, Pseudopseudohypoparathyroidism.

Correct Answer. a

Copyright © 2014 Delhi Academy of Medical Sciences, All Rights Reserved. 71/94
(152). True about ganglion

a. Common in volar aspect

b. Seen adjacent to tendon sheath

c. Never communicates with joint cavity

d. Most often found around the ankle joint

Solution. (b) Seen adjacent to tendon sheath


Ref: Adam’s 11/e, P 246
Sol :

- A ganglion cyst, also known as a bible cyst, is a swelling that often appears on or around joints and tendons in the hand or foot. The size
of the ganglion or cyst can vary over time. It is most frequently located around the dorsum of the wrist and on the fingers.
- Ganglion cysts are idiopathic, but presumably reflect a variation in normal joint or tendon sheath function. Cysts near joints are
connected to the joint and the leading theory is that a type of check valve forms that allows fluid out of the joint, but not back in. The cyst
contains clear fluid similar to, but thicker than, normal synovial fluid. They are most often found around the wrist joint, especially at the
scapholunate joint, which accounts for 80% of all ganglion cysts.
- Treatment
- Ganglion cyst of the hand with multiple cystic chambers containing glairy material. The walls are composed of bland fibrous tissue with
no specialized lining.
- With surgery, the recurrence rate is reduced to 5 to 10% if the check valve at the joint capsule is removed. Alternatively a needle may
be used to drain the fluid from the cyst (aspiration), however if the fluid has become thick due to the passage of time this treatment is not
always effective.

Correct Answer. b

(153). Which of the following is derived from the radial artery?

a. Dorsal and palmar carpal arteries

b. Common interosseous artery

c. Anterior interosseous artery

d. Poster interosseous artery

Solution. (a) Dorsal and palmar carpal arteries


Ref: Read the text below
Sol :
- The radial artery gives rise to the radial recurrent artery as well as dorsal and palmar carpal branches.
- The ulnar artery gives rise to the common interosseous artery, anterior and posterior interosseous arteries, anterior and posterior ulnar
recurrent arteries, and dorsal and palmar carpal branches.

Correct Answer. a

Copyright © 2014 Delhi Academy of Medical Sciences, All Rights Reserved. 72/94
(154). A 59-year-old woman with pain at the side of her skull comes to the emergency department. An emergent head computed tomography
(CT) scan shows a large lesion in the internal auditory meatus. This condition may progress and damage which of the following pairs of
structures?

a. Vagus and glossopharyngeal nerves

b. Internal carotid and vertebral arteries

c. Internal jugular vein and trigeminal nerve

d. Facial and vestibulocochlear nerves

Solution. (d) Facial and vestibulocochlear nerves


Ref:Read the text below
Sol:
- The internal auditory meatus transmits the facial and vestibulocochlear nerves.
- The jugular foramen transmits the glossopharyngeal, vagus, and accessory nerves and the internal jugular vein.
- The ophthalmic, maxillary, and mandibular divisions of the trigeminal nerve run through the superior orbital fissure, foramen
rotundum, and foramen ovale, respectively.
- The hypoglossal nerve runs through the hypoglossal canal.

Correct Answer. d

(155). After ingesting a toxic substance, a 12-year-old girl is unable to close her lips. Which of the following muscles may be paralyzed?

a. Levator labii superioris

b. Zygomaticus minor

c. Orbicularis oris

d. Lateral pterygoid

Solution. (c) Orbicularis oris


Ref:Read the text below
Sol:
- The lips are closed by the orbicularis oris muscles.
- The lips are opened by the levator labii superioris, zygomaticus minor, and depressor labii inferioris muscles.
- The lateral pterygoid muscle can open the mouth by depressing the lower jaw.

Correct Answer. c

(156). A patient with a stab wound receives a laceration of the musculocutaneous nerve. Which of the following conditions is most likely to have
occurred?

a. Lack of sweating on the lateral side of the forearm

b. Inability to extend the forearm

c. Paralysis of brachioradialis muscle

d. Loss of tactile sensation on the arm

Solution. (a) Lack of sweating on the lateral side of the forearm


Ref:Read the text below
Sol:
- The musculocutaneous nerve contains sympathetic postganglionic fibers that supply sweat glands and blood vessels on the lateral side
of the forearm as the lateral antebrachial cutaneous nerve.
- The musculocutaneous nerve does not supply the extensors of the forearm and the brachioradialis.
- This nerve also supplies tactile sensation on the lateral side of the forearm but not the arm and supplies blood vessels on the lateral side
of the forearm but not the hand.

Correct Answer. a

Copyright © 2014 Delhi Academy of Medical Sciences, All Rights Reserved. 73/94
(157). A 20-year-old man fell from the parallel bar ,aneurologic examination reveals that he has a lesion of the lateral cord of the brachial
plexus.
Which of the following muscles is most likely weakened by this injury?

a. Subscapularis

b. Teres major

c. Latissimus dorsi

d. Pectoralis major

Solution. (d) Pectoralis major


Ref:Read the text below
Sol:
- The pectoralis major is innervated by the lateral and medial pectoral nerves originating from the lateral and medial cords of the
brachial plexus, respectively.
- The subscapularis, teres major, latissimus dorsi, and teres minor muscles are innervated by nerves originating from the posterior cord
of the brachial plexus.

Correct Answer. d

(158). A 24-year-old carpenter suffers a crush injury of his entire little finger. Which of the following muscles is most likely to be spared?

a. Flexor digitorum profundus

b. Extensor digitorum

c. Palmar interossei

d. Dorsal interossei

Solution. (d) Dorsal interossei


Ref:Read the text below
Sol:
- The dorsal interossei are abductors of the fingers.
- The little finger has no attachment for the dorsal interosseous muscle
because it has its own abductor.
- Therefore, the dorsal interosseous muscle is not affected. Other muscles are attached to the little finger; thus, they are injured.

Correct Answer. d

(159). An elderly woman with osteoporosis is taken to the emergency department following a fall. One of her legs appears shortened and is
externally rotated. A fracture of which part of the femur is suggested by these findings?

a. Greater trochanter

b. Lateral epicondyle

c. Medical epicondyle

d. Neck

Solution. (d) Neck


Ref:Read the text below
Sol:
- This is the classic presentation of a fracture of the neck of the femur. This type of fracture typically occurs in postmenopausal women
with significant bone resorption due to osteoporosis.
- Dislocation of the head of the femur can produce a similar effect. The change in the position of the leg is due to the action of the gluteal
muscles, particularly the gluteus maximus.

Correct Answer. d

Copyright © 2014 Delhi Academy of Medical Sciences, All Rights Reserved. 74/94
(160). Clinically, gender is not apparent until approximately which week of embryonic life :

a. 8th

b. 10th

c. 12th

d. 14th

Solution. (c) 12th


Ref:Novak’s Gynecology 12th -92
Sol:
- Approximately the 12th week of embryonic life, the differentiation of the
gonads takes place.
- The process depends on the elaboration of testis-determining factor (TDF) and, subsequently, by androgens produced by the male
gonad.

Correct Answer. c

Copyright © 2014 Delhi Academy of Medical Sciences, All Rights Reserved. 75/94
(161). All are sphincters of lower genital tract of female except

a. Pubovaginalis

b. Internal urethral sphincter

c. External urethral sphincter

d. Bulbospongiosus

Solution. (d) Bulbospongiosus


Ref:Gray’s anatomy 40thed pg 109
Sol:
Bulbospongiosus differs between the sexes.
In the male It lies in the midline, anterior to the perineal body and consists of two symmetrical parts united by a median fibrous raphe.
The fibres attach to the perineal body, in which they decussate, and are
attached to the transverse superficial perinei and the external anal sphincter.
The fibres diverge like the halves of a feather from the median raphe.
A thin layer of posterior fibres unites with the posterior portion of the perineal membrane.
The majority of the middle fibres encircle the bulb of the penis and adjacent corpus spongiosum and attach to an aponeurosis on the
dorsal surfaces. The anterior fibres spread out over the sides of the corpora cavernosa, ending partly in them, anterior to
ischiocavernosus, and partly in a tendinous expansion which covers the dorsal vessels of the penis.
Actions
Bulbospongiosus helps to empty the urethra of urine after the bladder has emptied.
It may assist in the final stage of erection as the middle fibres compress the erectile
tissue of the bulb and the anterior fibres contribute by compressing the deep
dorsal vein of the penis.
It contracts six or seven times during ejaculation, assisting in the expulsion of semen. In the female Bulbospongiosus also attaches to the
perineal body, but the muscle on each side is separate and covers the superficial parts of the vestibular bulbs and greater vestibular
glands.
They run anteriorly on either side of the vagina to attach to the corpora cavernosa clitoridis.
A few fibres cross over the dorsum of the body of the clitoris.
The muscle acts to constrict the vaginal orifice and express the secretions of the greater vestibular glands. Anterior fibres contribute to
erection of the clitoris by compressing its deep dorsal vein. About Pubovaginalis
- The pubococcygeal part of Levator ani isattached to the back of the body of the
pubis and passes back almost horizontally.
- The most medial fibres run directly lateral to the urethra and its sphincter as it passes through the pelvic floor. In males
- These fibres therefore lie lateral and inferior to the prostate and are referred to as pubourethralis.
- They form part of the urethral sphincter complex together with the intrinsic striated and smooth musculature of the urethra and fibres
decussate across the midline directly behind the urethra.
In females
- The fibres of this part of the muscle run further back to from a sling around the posterior wall of the vagina and are referred to as
pubovaginalis.
- In both sexes fibres from this part of pubococcygeus attach to the perineal body and a few elements also attach to the anorectal
junction.
- Some of these fibres, sometimes called puboanalis, decussate and blend with the longitudinal rectal muscle and fascial elements to
contribute to the conjoint longitudinal coat of the anal canal.
- Behind the rectum some fibres of pubococcygeus form a tendinous
intersection as part of the levator raphe but a thick muscular sling, puborectalis, wraps around the anorectal junction.
- Some fibres blend with those of the external anal sphincter.
- Pubococcygeus is a lateral compressor of the various visceral canals which cross the pelvic floor.
- Levator Ani must relax appropriately to permit expulsion of urine and particularly faeces. Levator ani also forms much of the basin-
shaped muscular pelvic diaphragm, which supports the pelvic viscera and it contracts with abdominal muscles and the abdominothoracic
diaphragm to raise intra-abdominal pressure.

Correct Answer. d

Copyright © 2014 Delhi Academy of Medical Sciences, All Rights Reserved. 76/94
(162). Tear secretion is absent in injury to

a. Greater petrosal nerve

b. Nasociliary nerve

c. Superior orbital nerve

d. Maxillary nerve

Solution. (a) Greater petrosal nerve


Ref:Keith L. Moore 6thed pg 1059
Sol:

Correct Answer. a

Copyright © 2014 Delhi Academy of Medical Sciences, All Rights Reserved. 77/94
(163). A 32-year-old woman is evaluated of an “enlarged ovarian” with an ovarian cyst. The patient has a transvaginal ultrasound which
demonstrates a left ovary measuring 5 x 4 x 3 cm with a 1.1 cm single varian cyst. The right ovary is 4 x 3 x 3 cm. You should indicate to
her that the left ovary is:

a. Abnormal because it is approximate twice the size of a normal ovary

b. abnormal because it contains a cyst

c. Normal in size and the cyst is likely functional

d. Normal because ovaries can be twice the average anticipated size

Solution. (c) Normal in size and the cyst is likely functional


Ref:Novak’s Gynecology 12th -102
Sol:
- The normal sized ovary in a reproductive aged women varies in size but is about 5 x 3 x 3 cm, and frequently contains small functional
cysts.

Correct Answer. c

(164). The deep layer of the superficial perineal fascia, or Colles’ fascia, is continuous superiorly with the deep layer of the superficial
abdominal fascia, which is called the:

a. Deep transverse perineal fascia

b. Urogenital fascia

c. Scarpa’s fascia

d. Camper’s fascia

Solution. (c) Scarpa’s fascia


Ref:Novak’s Gynecology 12th -112
Sol:
- The deep layer of the superficial abdominal fascia is called Scarpa’s fascia, which attaches from the ischiopublic rami and ischial
tuberosities.
- The superficial perineal compartment is continuous superiorly with the
superficial fascial spaces of the anterior abdominal wall.
- Thus, blood and infection can spread along this route.

Correct Answer. c

Copyright © 2014 Delhi Academy of Medical Sciences, All Rights Reserved. 78/94
(165). The bile and pancreatic ducts enter which of the following structures?

a. Stomach

b. 2nd portion of the duodenum

c. Cecum

d. Ileum

Solution. (b) 2nd portion of the duodenum


Ref:Read the text below
Sol:
- The bile and pancreatic ducts enter the posteromedial wall of the 2nd portion of the duodenum

Correct Answer. b

(166). Omental appendices are located on which of the following structures?

a. Stomach

b. Duodenum

c. Ileum

d. Ascending colon

Solution. (d) Ascending colon


Ref: Essential Clinical Anatomy by Moore p 249).
Sol:
- The large intestine can be distinguished from the small intestine by teniae coli, haustra, and omental appendices

Correct Answer. d

Copyright © 2014 Delhi Academy of Medical Sciences, All Rights Reserved. 79/94
(167). Thereare no teniae coli in which of the following structures?

a. Ascending colon

b. Transverse colon

c. Sigmoid colon

d. Appendix

Solution. (d) Appendix


Ref:Essential Clinical Anatomy by Moore p 249).
Sol:
 The taenia coli (also teniae coli) are three separate longitudinal ribbons of
smooth muscle on the outside of the ascending, transverse, descending and
sigmoid colons.
- They are visible, and can be seen just below the serosa or fibrosa.
- They are the Mesocolic, Free and Omental Coli. The teniae coli contracts
length wise to produce the haustra, the bulges in the colon.
- There are no teniae in the appendix or rectum

Correct Answer. d

(168). The high death rate associated with breast cancer is related to which of the following?

a. Nerve supply

b. Blood supply

c. Venous drainage

d. Complex lymphatic drainage

Solution. (d) Complex lymphatic drainage


Ref:Essential Clinical Anatomy by Moore-78
Sol:
- Because the axillary lymph nodes are the most common site of metastases from a breast cancer, enlargement of the palpable nodes in a
woman suggests the possibility of breast cancer and may be key to early detection.
- However, the absence of enlarged axillary nodes is no guarantee that metastasis from a breast cancer has not occurred, because the
malignant cells may have passed to other nodes, such as the infraclavicular and supraclavicular lymph nodes

Correct Answer. d

(169). The median nerve does which of the following?

a. Innervates the elbow joint with articular branches

b. Innervates the medial half of the flexor digitorum profundus

c. Innervates the hypothenar muscles

d. Innervates lumbricals 3 and 4

Solution. (a) Innervates the elbow joint with articular branches


Ref: Read the text below
Sol :
- The median nerve assists in the innervations of the elbow joint and gives
muscular branches to pronator teres, flexor carpi radialis, Palmaris longus, and flexor digitorum superficialis.
- The median nerve also has an anterior interosseous branch that innervates the lateral part of the flexor digitorum profundus, flexor
pollicis longus, and pronator quadratus.
- The recurrent branch of the median nerve innervates the thenar muscles, and the palmar cutaneous branch innervates the skin of the
lateral part of the palm.

Correct Answer. a

Copyright © 2014 Delhi Academy of Medical Sciences, All Rights Reserved. 80/94
(170). Which of the following is not true in respect to the flexor pollicis brevis?

a. It flexes the thumb at the carpometacarpal joint.

b. It flexes the thumb at the metacarpophalangeal joint.

c. Its tendon typically contains a sesamoid bone.

d. It is innervated by C5–C6.

Solution. (d) It is innervated by C5–C6.


Ref: Read the text below
Sol :
- The flexor pollicis brevis is located medial to the abductor pollicis brevis.
- It flexes the thumb at the carpometacarpal and metacarpophalangeal joints and assists in opposition. Its tendon typically contains a
sesamoid bone.
- It is innervated by the recurrent branch of the median nerve (C8–T1)

Correct Answer. d

(171). A 71-year-old woman has opacification of the structure identified in Image G. Which of the following underlying diseases is she most
likely to
have?

a. Marfan syndrome

b. Diabetes mellitus

c. Hypercholesterolemia

d. Rheumatoid arthritis

Solution. (b) Diabetes mellitus


Ref:Read the text below
Sol :
- This is the crystalline lens of the eye, and cataract formation is not
uncommon in older persons.
- In persons with diabetes mellitus, this process is more likely to occur. The cataract reduces vision, but it can be removed surgically.

Correct Answer. b

Copyright © 2014 Delhi Academy of Medical Sciences, All Rights Reserved. 81/94
(172). Which of the following is correct regarding the triangle of auscultation?

a. Its borders are the latissimus dorsi, scapula, and trapezius.

b. It is a good location to hear heart murmurs.

c. The 8th and 9th ribs and the 8th intercostals space are subcutaneous here.

d. It is a location of back trauma.

Solution. (a) Its borders are the latissimus dorsi, scapula, and trapezius.
Ref: Read the text below
Sol :
- The triangle of auscultation, a good place to examine lung sounds, is bounded by the superior horizontal border of the latissimus dorsi,
the medial border of the scapula, and the inferolat- eral border of the trapezius.
- The 6th and 7th ribs and the 6th intercostal space is subcutaneous

Correct Answer. a

(173). All of the following structures are located within the rectus sheath except

a. Pyramidalis

b. Rectus abdominis

c. Inferior epigastric arteries and veins

d. Deep inguinal ring

Solution. (d) Deep inguinal ring


Ref: Read the text below
Sol :
- The contents of the rectus sheath include the pyramidalis and rectus abdominis muscles, the superior and inferior epigastric arteries
and veins, the lymphatics, and ventral primary rami of T7–T12 nerves(Moore, p 180).

Correct Answer. d

(174). Inferiorly, the inferior margin of the external oblique aponeurosis thickens and folds back on itself to form which of the following
structures?

a. Rectus sheath

b. Inguinal ligament

c. Arcuate line

d. Deep inguinal ring

Solution. (b) Inguinal ligament


Ref: Read the text below
Sol :
- Inferiorly, the inferior margin of the external oblique aponeurosis thickens and folds back on itself to form the inguinal ligament, a
fibrous band extending between the anterior superior iliac spine and the pubic tubercle.

Correct Answer. b

Copyright © 2014 Delhi Academy of Medical Sciences, All Rights Reserved. 82/94
(175). Compression of the lateral branch of posterior interosseous nerve syndrome causes paralysis of all except :

a. Extensor pollicis brevis

b. Extensor pollicis longus

c. Extensor indicis proprius.

d. Extensor carpi ulnaris

Solution. (d) Extensor carpi ulnaris


Ref: Marcer’s orthopaedic surgery 9/e, p-859-891
Sol :
- The etiology of posterior interosseous nerve syndrome is similar to that of radial tunnel syndrome.
- Compression is thought to occur after takeoff of the branches to the radial wrist extensors and RSN.
- After emerging from the supinator, the nerve may be compressed before it bifurcates into medial and lateral branches, causing a
complete paralysis of the digital extensors and dorsoradial deviation of the wrist secondary to paralysis of the extensor carpi ulnaris.
- If compression occurs after the nerve bifurcates, selective paralysis of muscles occurs, depending on which branch is involved.
Compression of the medial branch causes paralysis of the extensor carpi ulnaris, extensor digiti, and extensor digitorum communis.
Compression of the lateral branch causes paralysis of the abductor pollicis longus, extensor pollicis brevis, extensor pollicis longus, and
extensor indicis proprius.
- Most commonly, entrapment occurs at the proximal edge of the supinator. Other possible etiologies for posterior interosseous nerve
dysfunction include trauma (Monteggia fractures), synovitis (rheumatoid), tumors, and iatrogenic injuries.

Correct Answer. d

(176). Tufting of distal phalanx is seen in :

a. Gout

b. Pseudogout

c. Psoriasis

d. Hyperparathyroidism

Solution. (d) Hyperparathyroidism


Ref: Read the text below
Sol :
- Primary hyperparathyroidism is the unregulated overproduction of PTH
resulting in abnormal calcium homeostasis. Symptoms related to
hyperparathyroidism may include bone pain, pathologic fractures, and
nephrolithiasis.
- Symptoms related to hypercalcemia may include muscle weakness. Volume depletion, polyuria and polydipsia, neuropsychiatric
symptoms, peptic ulcer disease, and pancreatitis.

Correct Answer. d

Copyright © 2014 Delhi Academy of Medical Sciences, All Rights Reserved. 83/94
(177). “Tennis elbow is characterized by :

a. Tenderness over the medial epicondyle

b. Tendinitis of common extensor origin

c. Tendonitis of common flexor origin

d. Painful flexion and extension

Solution. (b) Tendinitis of common extensor origin


Ref: Read the text below
Sol :
Lateral epicondylitis or lateral epicondylalgia, also known as tennis elbow, shooter's elbow and archer's elbow, is a condition where the
outer part of the elbow becomes sore and tender. It is commonly associated with playing tennis and other racquet sports Signs and
symptoms
- Pain on the outer part of elbow (lateral epicondyle).
- Point tenderness over the lateral epicondyle – a prominent part of the bone on the outside of the elbow.
- Gripping and movements of the wrist hurt, especially wrist extension[citation needed] and lifting movements.
- Activities that use the muscles that extend[citation needed] the wrist (e.g. pouring a pitcher or gallon of milk, lifting with the palm down) are
characteristically painful.
- Morning stiffness.

Correct Answer. b

(178). The reflected inguinal ligament receives fibers from the contralateral aponeurosis of which of the following structures?

a. External abdominal oblique

b. Internal abdominal oblique

c. Transverse abdominal oblique

d. Rectus abdominis

Solution. (a) External abdominal oblique


Ref: Read the text below
Sol :
- Some fibers of the inguinal ligament pass upward to cross the linea alba and blend with the lower fibers of the contralateral
aponeurosis.
- These fibers form the reflected inguinal ligament

Correct Answer. a

Copyright © 2014 Delhi Academy of Medical Sciences, All Rights Reserved. 84/94
(179). Between the internal oblique and transverse abdominal muscles is a neurovascular plane that contains all of the following except

a. Iliohypogastric nerve

b. Deep circumflex iliac artery

c. Inferior epigastric artery

d. Subcostal nerve

Solution. (c) Inferior epigastric artery


Ref: Read the text below
Sol :
- The neurovascular plane of the anterolateral abdominal wall contains the thoracoabdominal nerves, cutaneous branches T7–T11,
ventral ramus of T12, iliohypogastric and ilioinguinal nerves, lumbar arteries, and the deep circumflex iliac artery .

Correct Answer. c

(180). The rectus abdominis muscle is anchored transversely by attachment to the anterior layer of the rectus sheath by which of the following
structures?

a. Pubic tubercle

b. Xiphoid process

c. Linea alba

d. Tendinous intersections

Solution. (d) Tendinous intersections


Ref: Read the text below
Sol :
- The rectus muscle is anchored transversely by attachments to the anterior layer of the rectus sheath and three or more tendinous
intersections.

Correct Answer. d

(181). All of the following bones contribute to the formation of the pelvic cavity except

a. Ischium

b. Pelvis

c. Pubis

d. Coccyx

Solution. (d) Coccyx


Ref: Read the text below
Sol :
- The pelvic girdle is formed by the hip (ischium, pubis, and ilium) bones and the sacrum

Correct Answer. d

Copyright © 2014 Delhi Academy of Medical Sciences, All Rights Reserved. 85/94
(182). Spinal part of accessory nervesupplies :

a. Pectoralis major

b. Pectoralis minor

c. Sternocleidomastoid

d. Deltoid

Solution. (c) Sternocleidomastoid


Ref: BDC - 335
Sol :
- Accessory nerve eleventh cranial nerve; origin, by cranial roots from the side of the medulla oblongata, and by spinal roots from the
side of the spinal cord (from the upper three or more cervical segments); the roots unite to form the trunk of the accessory nerve, which
divides into an internal branch (cranial portion) and an external branch (spinal portion); distribution, the internal branch to the vagus
and thereby to the palate, pharynx, larynx, and thoracic viscera; the external branch branches to the sternocleidomastoid and trapezius
muscles; modality, parasympathetic and motor.

Correct Answer. c

(183). In neonates, spinal cord ends at :

a. L1

b. L2

c. L3

d. L4

Solution. (c) L3
Ref: Clinical Anatomy-Snell, 8th ed., Pg. 867
Sol :
- The spinal cord ends at the level of the L1 or 2 vertebra. The range is from one vertebra higher to one vertebra lower: T12 to L3 .
- In the newborn, the spinal cord extends to L3 vertebra.

Correct Answer. c

Copyright © 2014 Delhi Academy of Medical Sciences, All Rights Reserved. 86/94
(184). Inferior oblique muscle of eye is supplied by :

a. Oculomotor nerve

b. Abducent nerve

c. Trochlear nerve

d. Facial nerve

Solution. (a) Oculomotor nerve


Ref: Read the text below
Sol :
- Inferior oblique muscle of eye is supplied by Oculomotor nerve.

Correct Answer. a

(185). The trapezoid body is concerned with :

a. Pain and temperature

b. Hearing

c. touch and pressure

d. Proprioception

Solution. (b) Hearing


Ref: Read the text below
Sol :
- The trapezoid body is concerned with Hearing.

Correct Answer. b

Copyright © 2014 Delhi Academy of Medical Sciences, All Rights Reserved. 87/94
(186). Lateral rotation and extension of the thigh is limited by :

a. Ischiofemoral ligaments

b. Pubofemoral ligaments

c. Iliofemoral ligaments

d. Sacroiliac ligament

Solution. (c) Iliofemoral ligaments


Ref: Clinical Anatomy-Snell, 8th ed., Pg. 589
Sol :
The movements of the hip joint is thus performed by a series of muscles which are here presented in order of importance[ with the range
of motion from the neutral zero-degree position indicated:
- Lateral or external rotation (30° with the hip extended, 50° with the hip flexed): gluteus maximus; quadratus femoris; obturator
internus; dorsal fibers of gluteus medius and minimus; iliopsoas (including psoas major from the vertebral column); obturator externus;
adductor magnus, longus, brevis, and minimus; piriformis; and sartorius. The iliofemoral ligament inhibits lateral rotation and extension,
this is why the hip can rotate laterally to a greater degree when it is flexed.
- Medial or internal rotation (40°): anterior fibers of gluteus medius and minimus; tensor fascia latae; the part of adductor magnus
inserted into the adductor tubercle; and, with the leg abducted also the pectineus.
- Extension or retroversion (20°): gluteus maximus (if put out of action, active standing from a sitting position is not possible, but
standing and walking on a flat surface is); dorsal fibers of gluteus medius and minimus; adductor magnus; and piriformis. Additionally,
the following thigh muscles extend the hip: semimembranosus, semitendinosus, and long head of biceps femoris. Maximal extension is
inhibited by the iliofemoral ligament.
- Flexion or anteversion (140°): the hip flexors: iliopsoas (with psoas major from vertebral column); tensor fascia latae, pectineus,
adductor longus, adductor brevis, and gracilis. Thigh muscles acting as hip flexors: rectus femoris and sartorius. Maximal flexion is
inhibited by the thigh coming in contact with the chest.
- Abduction (50° with hip extended, 80° with hip flexed): gluteus medius; tensor fascia latae; gluteus maximus with its attachment at the
fascia lata; gluteus minimus; piriformis; and obturator internus. Maximal abduction is inhibited by the neck of the femur coming into
contact with the lateral pelvis. When the hips are flexed, this delays the impingement until a greater angle.
- Adduction (30° with hip extended, 20° with hip flexed): adductor magnus with adductor minimus; adductor longus, adductor brevis,
gluteus maximus with its attachment at the gluteal tuberosity; gracilis (extends to the tibia); pectineus, quadratus femoris; and obturator
externus. Of the thigh muscles, semitendinosus is especially involved in hip adduction. Maximal adduction is impeded by the thighs
coming into contact with one another. This can be avoided by abducting the
opposite leg, or having the legs alternately flexed/extended at the hip so they travel in different planes and do not intersect.

Correct Answer. c

(187). A 36-year-old woman is hospitalized for treatment of a stomach ulcer. Radiographic studies reveal ulcer along the greater curvature,
approximated, 4 cm away from the pyloric sphincter. Surgery reveals that the ulcer has eroded through the stomach wall and has
damaged the artery supplying the involved region of the stomach. Which artery was likely involved?

a. Left gastric

b. Left gastroepiploic

c. Right gastric

d. Right gastroepiploic

Solution. (d) Right gastroepiploic


Ref: Read the text below
Sol:
- The right gastroepiploic artery, off the gastroduodenal artery, supplies the right half of the greater-urvature of the stomach and could
be directly affected by ulceration of the greater curvature of the stomach at a site this close (4 cm) to the pyloric sphincter.

Correct Answer. d

Copyright © 2014 Delhi Academy of Medical Sciences, All Rights Reserved. 88/94
(188). A 49-yrs-old man is diagnosed with tuberculosis. On physical examination, large flocculent masses are noted over the lateral lumbar
back,
and a similar mass is located in the ipsilateral groin. This pattern of involvement strongly suggests an abscess tracking along which of
the following muscles?

a. Adductor longus

b. Gluteus maximu

c. Gluteus minimus

d. Psoas major

Solution. (d) Psoas major


Ref: Read the text below
Sol:
- This is the classic presentation of a psoas abscess. This clinical entity was formerly a fairly common complication of vertebral
tuberculosis, but is now rare in clinical practice in this country.
- The psoas muscle is covered by a fibrous sheath known as the psoas fascia. This sheath is open superiorly, permitting an infection
involving the soft tisues around the spine to enter the sheath, the track down to the groin.
- The adductor longus is a muscle of the medial thigh and is not related to the lumbar portion of the back.

Correct Answer. d

(189). A 24-year-old man is stabbed in the neck resulted in left-sided hemiparesis. There is loss of discriminatory joint position and vibration
sensation below C8 on the left side. On the right side, there is loss of pain and temperature sensation below C8.MRI of the cervical spinal
cord will show ?

a. A complete transaction of the spinal cord

b. A hemisection of the left side of the cord

c. A lesion of the dorsal columns of the cord on the left side

d. A lesion of the lateral funiculus on the left side

Solution. (b) A hemisection of the left side of the cord


Ref: Read the text below
Sol:
- A hemisection of the cord produces Brown-Sequard syndrome. Below the level of the lesion there is ipsilateral hemiparesis, ipsilateral
loss of discriminatory touch sensation, and contralateral loss of pain and temperature sensation.
- A hemisection of the left half of the spinal cord will transect the fibers of the corticospinal tract on the left side, resulting in left-sided
motor paralysis. (These motor fibers originate in the contralateral cortex, decussate in the medulla, and descend in the lateral funiculus
of the spinal cord).
- The hemisection will also transect the ascending fibers of the dorsal
column pathway that carry sensory information concerning discriminatory touch, vibration, and joint position from the same side. (This
tract will subsequently cros in the medulla and in the contralateral cerebral cortex).
- In contrast, the fibers carrying information about pain and temperature
decussate segmentally in the spinal cord and form the spinothalamic tract, which ascends in the lateral funciulus.
- Damaging the spinothalamic tract on the left side impairs sensation arising from the right side of the body.

Correct Answer. b

Copyright © 2014 Delhi Academy of Medical Sciences, All Rights Reserved. 89/94
(190). A 47-year-old man presents with an unsteady, broad-based gait and slow, slurred speech. Neurologic examination reveals
dysdiadochokinesia,
intention tremor, hypotonia, and nystagmus. The patient’s lesion is in a brain structure that derives from which of the following
embryonic structures?

a. Diecephalon

b. Mesencephalon

c. Metencephalon

d. Myelencephalon

Solution. (c) Metencephalon


Ref: Read the text below
Sol:
- The patient has a cerebellar lesion. Cerebellar dysfunction can lead to a
variety of motor dysfunctions, including truncal ataxia (appearing similar to the gait of an intoxicated individual), intention tremor
(uncontrolled shaking of affected extremity present only with purposeful movement), dysdiadochokinesia (the inability to perform rapid
and regular alternating movements), dysmetria (inability to stop movements at the desired point), dysarthria (ataxic speech), hypotonia,
and nystagmus.

Correct Answer. c

(191). The mitral valve can be best evaluated by auscultation at which of the following locations?

a. Left second intercostals space

b. Left fifth intercostals space

c. Left lower sterna body border

d. Right second intercostals space

Solution. (b) Left fifth intercostals space


Ref: Read the text below
Sol:
- The mitral valve is most audible over the left fifth intercostals space at the midclavicular line.

Correct Answer. b

(192). Which of the following structures is located between the ischial spine and the ischial tuberosity?

a. Obturator foramen

b. Lesser sciatic notch

c. Acetabular notch

d. Pubic arch

Solution. (b) Lesser sciatic notch


Ref: Read the text below
Sol :
- The concavity between the ischial spine and the ischial tuberosity is the
lesser sciatic notch.
- The larger concavity, the greater sciatic notch, is superior to the ischial spine and is formed in part by the ilium (Moore, p 334).

Correct Answer. b

Copyright © 2014 Delhi Academy of Medical Sciences, All Rights Reserved. 90/94
(193). Which of the following nerves exits the pelvis through the greater sciatic foramen, superior to the piriformis?

a. Sciatic

b. Pudendal

c. Superior gluteal

d. Lumbosacral trunk

Solution. (c) Superior gluteal


Ref: Read the text below
Sol :
- The superior gluteal nerve arises from the posterior divisions of the ventral rami of L4 through S1 and leaves the pelvis through the
greater sciatic foramen, superior to the piriformis

Correct Answer. c

(194). The inferior alveolar nerve block is the most common type of nerve block used for dental procedures. It is a branch of which part of
mandibular nerve?

a. Anterior division

b. Posterior division

c. main trunk

d. Direct continuation

Solution. (b) Posterior division


Ref.:Read the text below
Sol :
- The inferior alveolar nerve (sometimes called the inferior dental nerve) is a branch of the mandibular nerve, which is itself the third
branch (V3) of the trigeminal nerve (cranial nerve V).

Correct Answer. b

(195). The thalamostriate vein & choroid vein unite to form:

a. Great cerebral vein

b. Middle cerebral vein

c. Internal cerebral vein

d. Basal vein

Solution. (c) Internal cerebral vein


Ref.:Read the text below
Sol :
- The internal cerebral veins are paired, parmedian veins which course posteriorly along the roof of the third ventricle, between the two
leaves of the velum interpositum.
- Each is formed at the foramen of Monro by the confluence of the choroidal vein (draining the choroid plexus of the lateral ventricle),
and the thalamostriate vein (which lies in the groove between the thalamus and caudate nucleus and receives blood from both). The
veins of the septum pellucidum usually join the thalamostriate vein.
- The internal cerebral veins unite with the basal veins (of Rosenthal) to form the great cerebral vein (of Galen) just beneath the splenium
of the corpus callosum in the quadrigeminal cistern. The confluence of the great cerebral vein and inferior sagittal sinus forms the
straight sinus.

Correct Answer. c

Copyright © 2014 Delhi Academy of Medical Sciences, All Rights Reserved. 91/94
(196). True about the location of sphenoidal emissary formen is:

a. Medial to foramen ovale

b. Between foramen ovale & foramen spinosum

c. Posterolateral to foramen ovale

d. Between sphenoid & apex of petrous temporal bone

Solution. (a) Medial to foramen ovale


Ref.:Read the text below
Sol :
- Sphenoidal Emissary Foramen (Foramen of Vesalius)
- Foramen present between scaphoid fossa & foramen ovale (medial to foramen ovale).
- It transmits an emissary vein connecting cavernous sinus with pterygoid venous plexus.

Correct Answer. a

(197). Chamberlain line joins?

a. Palate to occiput

b. Palate to temporal bone

c. Palate to foramen magnum

d. Palate to parietal bone

Solution. (c) Palate to foramen magnum


Ref.:Read the text below
Sol :

Correct Answer. c

Copyright © 2014 Delhi Academy of Medical Sciences, All Rights Reserved. 92/94
(198). A 43-year-old man has had frequent headaches for the past 2 months. He has visual field defects that limit his lateral vision. MR imaging
reveals a lesion located in the structure identified in Image H. Which of the following laboratory test findings is most likely to be present
in this man?

a. Hyperprolactinemia

b. Hypernatremia

c. Hypertriglyceridemia

d. Hypergammaglobulinemia

Solution. (a) Hyperprolactinemia


Ref:Read the text below
Sol :
- The anterior pituitary gland in the sella turcica can have adenomas arise within it.
- Such adenomas can have both pressure effects (on the surrounding bone to produce headache and on the optic chiasm to produce
bitemporal
hemianopsia) from enlargement and hormonal effects.
- The most common hormonal production is prolactin.

Correct Answer. a

(199). Dawbarn’s sign is due to involvement of which of the following?

a. Infrapatellar bursa

b. Prepatelar bursa

c. Sub acromial bursa

d. Sub scapularis bursa

Solution. (c) Sub acromial bursa


Ref.:Read the text below
Sol :
Subacromial Bursitis
- In sub acromial bursitis pressure over the deltoid below the acromion with arm by side causes pain.
- When arm is abducted pressure over the same point causes no pain, because the bursae disappears under the acromion (Dawbarn’s
sign)
- Usually occurs secondary to impingement/tendonitis of rotoator cuff
tendons.

Correct Answer. c

Copyright © 2014 Delhi Academy of Medical Sciences, All Rights Reserved. 93/94
(200). Thoracodorsal nerve is a branch of:

a. Posterior cord of brachial plexus

b. Medial cord of brachial plexus

c. Lateral cord of brachial plexus

d. trunk of brachial plexus

Solution. (a) Posterior cord of brachial plexus


Ref.:Read the text below
Sol :
Thoracodorsal nerve
- Supplies latissimus dorsi
- Branch of posterior cord of brachial plexus

Correct Answer. a

Test Answer
1.(a) 2.(d) 3.(b) 4.(c) 5.(c) 6.(b) 7.(b) 8.(c) 9.(b) 10.(a)

11.(c) 12.(d) 13.(b) 14.(d) 15.(a) 16.(d) 17.(d) 18.(b) 19.(b) 20.(a)

21.(c) 22.(c) 23.(a) 24.(a) 25.(a) 26.(c) 27.(d) 28.(c) 29.(a) 30.(a)

31.(a) 32.(d) 33.(c) 34.(a) 35.(d) 36.(c) 37.(b) 38.(c) 39.(b) 40.(d)

41.(c) 42.(a) 43.(b) 44.(b) 45.(c) 46.(d) 47.(b) 48.(b) 49.(b) 50.(b)

51.(d) 52.(d) 53.(a) 54.(c) 55.(a) 56.(c) 57.(b) 58.(b) 59.(d) 60.(b)

61.(d) 62.(a) 63.(b) 64.(d) 65.(c) 66.(c) 67.(b) 68.(a) 69.(b) 70.(b)

71.(b) 72.(c) 73.(b) 74.(d) 75.(a) 76.(c) 77.(b) 78.(d) 79.(d) 80.(c)

81.(d) 82.(b) 83.(d) 84.(d) 85.(b) 86.(d) 87.(d) 88.(a) 89.(a) 90.(b)

91.(a) 92.(a) 93.(a) 94.(b) 95.(a) 96.(d) 97.(c) 98.(b) 99.(b) 100.(b)

101.(a) 102.(c) 103.(d) 104.(a) 105.(c) 106.(a) 107.(a) 108.(a) 109.(c) 110.(c)

111.(a) 112.(b) 113.(d) 114.(b) 115.(d) 116.(d) 117.(a) 118.(a) 119.(d) 120.(b)

121.(b) 122.(b) 123.(a) 124.(a) 125.(d) 126.(a) 127.(d) 128.(a) 129.(d) 130.(c)

131.(a) 132.(c) 133.(b) 134.(c) 135.(a) 136.(c) 137.(b) 138.(b) 139.(c) 140.(d)

141.(a) 142.(b) 143.(b) 144.(c) 145.(a) 146.(d) 147.(d) 148.(a) 149.(a) 150.(d)

151.(a) 152.(b) 153.(a) 154.(d) 155.(c) 156.(a) 157.(d) 158.(d) 159.(d) 160.(c)

161.(d) 162.(a) 163.(c) 164.(c) 165.(b) 166.(d) 167.(d) 168.(d) 169.(a) 170.(d)

171.(b) 172.(a) 173.(d) 174.(b) 175.(d) 176.(d) 177.(b) 178.(a) 179.(c) 180.(d)

181.(d) 182.(c) 183.(c) 184.(a) 185.(b) 186.(c) 187.(d) 188.(d) 189.(b) 190.(c)

191.(b) 192.(b) 193.(c) 194.(b) 195.(c) 196.(a) 197.(c) 198.(a) 199.(c) 200.(a)

Copyright © 2014 Delhi Academy of Medical Sciences, All Rights Reserved. 94/94

S-ar putea să vă placă și